NCLEX -Practice questions

¡Supera tus tareas y exámenes ahora con Quizwiz!

The nurse provides care to a patient diagnosed with COPD. Which goal is most appropriate to include on the care plan for this patient? A) Improve gas exchange B) Perform activities of daily living without dyspnea C) Obtain flu & pneumonia vaccinations D) Sleep for 8 hours without interruption

A) — Gas exchange is a priority for this client. B) Performing activities of daily living without dyspnea may be appropriate after gas exchange is improved. C) Vaccinations would be appropriate after gas exchange is improved. D) Sleep needs vary. This may not be realistic for this client.

The nurse provides care to a client diagnosed with acute pancreatitis. Which intervention will the nurse include in the client's care plan? (Select all that apply.) A) Monitor signs of hypocalcemia B) Observe for manifestations of respiratory infection C) Provide a diet low in carbohydrates D) Position side lyding with the head elevated 45 degrees E) Provide a diet high in fats

A, B, D, - {C} The diet should be high in carbohydrates because carbohydrates are less stimulating to the pancreas, {E}The diet should be restricted in fats because fat stimulates the release of cholecystokinin, which can aggravate an irritated pancreas.

What are the 5 P's of compartment syndrome? (SATA) A) Pain B) Paresthesia C) Parotitis D) PE E) Pulselessness F) Pallor G) Peritonitis H) Paralysis

A, B, E, F, H

The nurse provides care to a patient diagnosed with C.diff. Which precaution will the nurse take? (SATA) A) Wear a protective gown when entering the patient's room B) Wear a surgical respirator mask while administering medications C) Wear gloves when handling the food tray D) Patients visitors should wear a mask while in the patient's room E) Don surgical gloves with removing surgical dressing

A,B - C. Diff is contact precautions, gown & gloves; respirator is for airborne, a mask is not necessary, to remove a surgical dressing wear disposable gloves and then put on surgical gloves to clean dressing

The nurse is getting a 22 year old patient ready before surgery. What would the nurse delegate to the nursing assistive personnel? (SATA) A) Getting a clean catch urinalysis B) Taking vital signs C) Assess lung sounds D) Put on T.E.D. stockings E) Educate patient on correct technique to use incentive spirometer

A,B,D

The nurse plans a diet-teaching session for a chronic kidney disease patient. The nurse determines more teaching is needed when the patient selects which option? (SATA) A) Grilled mozzarella sandwich, canned veggies soup, diet cola B) Toast with peanut butter, banana and coffee C) Chicken sandwich with mustard, green salad, and lemonade D) Macaroni with salt substitute, pudding with raisins E) Spaghetti with meatballs, cabbage, and apple pie

A,B,D

The nurse provides care for the patient receiving radiation therapy for lung cancer. Which patient statement indicates to the nurse that further intervention is needed due to the side effects of radiation? (SATA) A) "I'm having difficulty swallowing" B) "I am missing days at work recently" C) "I haven't had a bowel movement in 4 days" D) "I have rash on my arm pits" E) "My hair is falling out in clumps"

A,B,D - Radiation to lungs could be affecting tissues around the throat {airway}, the patient is fatigued, radiation therapy can cause rashes → deodorant is an irritant; constipation and alopecia is usually found in CHEMO pts

While administering blood for a patient, which action will the nurse take to maintain safety? (SATA) A) Verify patient's identification with at least two identifiers B) Give the blood unit as soon as it arrives on the unit C) Administer the blood unit within 2 hours for increased benefit D) During the first 15 minutes, stay with the patient E ) Get a PRN order for 1-2 liters of oxygen during the transfusion time

A,B,D - verify identification with at least 2 identifiers, The RN should check vital signs every 5 minutes for the first 15 minutes, Blood should be administered within the first 30 minutes of the blood leaving the blood bank; blood should be given over 2-4 hours to reduce the risk of fluid overload, o2 is not routinely administered during blood transfusion

A nurse caring for a 40 year old woman with a history of urinary tract infections. Which strategies prevent UTIs?(SATA) A) Always wiping from front to back B) Voiding every 3-4 hours C) Stay hydrated with at least 6 glasses of water D) Take antibiotics until symptoms subside E) Refrain from taking bubble baths

A,B,D,E

The nurse is looking to prevent venous thromboembolism. Which action does the nurse delegate to the NAP? (SATA) a) Change the patient's position every 2 hours B) Help the patient ambulate as needed C) Give the patient education materials on venous thromboembolism D) Apply sequential compression devices E) Apply compression stockings

A,B,D,E

The nurse's updating the plan of care of a patient who had a hysterectomy. Which intervention should the nurse include in the plan of care? {SATA} A) Repositioning every 2 hours B) Assess for adequate urine output C) Fasting for 16 hours D) Assess for vaginal bleeding E) Schedule periods for walking and leg extensions

A,B,D,E

The nurse provides discharge instructions to an adult patient hospitalized for pneumococcal pneumonia. Which instruction does the nurse include in the teaching plan? (SATA) A) Complete the whole 7 days of antibiotics even if symptoms subside B) Keep using spirometer and performing the breathing exercises C) Inform your physician with any cold like symptoms D) Refrain from sitting with large groups of people because of low immunity E) Inform your doctor with signs of shortness of breath

A,B,D,E - cold like symptoms are to be expected, avoiding large crows will lower risk of being exposed to respiratory infection

The nurse on a med surg unit is giving discharge teaching for a patient treated for Clostridium Difficile. Which instructions does the nurse highlight as "call your doctor immediately category"? {SATA) A) Develop dry skin B) Increased heart rate > 105 bpm C) Stomach cramps D) Clammy limbs E) Polydipsia

A,B,D,E - these are signs of dehydration; {C} you will have cramps if diarrhea occurs

Patient is prescribed glimepiride for his type 2 diabetes mellitus. The nurse is reconciling the patient's medication, which entry in the patient's medication list causes the nurse to question the glimepiride prescription?(SATA) A) Atenolol B) Gemfibrozil C) Ginkgo biloba D) Ginseng E) Ibuprofen F) Valerian

A,B,D,E -Atenolol (Beta blocker) mask side effects of hypoglycemia, Gemfibrozil (cholesterol med) ^ hypoglycemic effect , Ginseng ^ hypoglycemia effect of sulfonylureas, Ibuprofen ^ hypoglycemic effects

The nurse delegates tasks to a nursing assistive personnel for the first time. Which actions are appropriate for the nurse to implement? (Select all that apply) A) Explain to the NAP which tasks will be delegated B) Explain to the NAP how to report task completion C) Permit the NAP to decide which patient changes to report D) Allow the NAP to decide when tasks will be completed E) Explain to the NAP the expected task outcomes

A,B,E

The nurse is giving dietary education to a patient diagnosed with Cushing syndrome. Which statements by the patient cause the nurse to take action? (SATA) A) I should follow a low protein diet B) I will double my carbohydrate intake C) I will include bananas and broccoli with every meal D) I should use a salt substitute with my meals E) I should increase my total daily calories

A,B,E

The nurse reviews care needed for a patient 2 days post-operation.Which would the nurse delegate to nursing assistive personnel (NAP)?(Select all that apply) A) Apply T.E.D. stockings B) Measure height and weight and document it C) Educate on performing breathing exercise D) Check for urinary retention using a bladder ultrasound E) Assist in wearing a leg prosthesis

A,B,E

The nurse provides care to a newly diagnosed patient with schizophrenia who is prescribed chlorpromazine 25 mg PO TID. Which patient statement indicates a correct understanding of taking the medication? (SATA) A) I'll brush my teeth three times a day B) It will take 4-6 weeks for my medication to start working C) I may have a glass of wine each night because it is good for the heart D) I can drive myself to work on Monday E) I have to get blood levels regularly for a couple of months F) I will not worry about pink urine

A,B,E,F - Chlorpromazine has a side effect of extremely dry mouth so oral hygiene is very important, phenothiazines take 4-6 weeks to see signs of improvement,chlorpromazine has an effect on white blood cells and can cause leukocytopenia, this medication is known to cause pink tinged urine; no alcohol, no heavy machinery {cars} on an antipsychotic,

Roles and responsibilities of the nurse manager position include: (SATA) A) Monitoring the professional practice model on the unit B) Coordinating the patient discharge care C) Empower the nursing staff at the administration's meetings D) Following up on patient's and family complaints E) Following up on the root cause analysis of incident reports

A,C,D

What potential long-term effects of fetal alcohol syndrome does the nurse include in teaching for pregnant women about alcohol consumption during pregnancy? (SATA) A) Delayed physical growth B) Facial deformities C) Respiratory depression D) Learning disabilities E) Hypotension

A,C,D

A 29 year old patient presented to the ED with dry cough, hyperthermia, chest crackles, severe headache, and muscle weakness. Which nursing diagnoses are appropriate for the nurse to include in the plan of care? {SATA} A) Discomfort B) Risk for aspiration C) Inefficient gas exchange D) Ineffective breathing pattern E) Risk for infection

A,C,D - {B} this pt is not at risk for aspiration, {E} this pt is not at risk for infection → they already have an infection based on their symptoms (actual outweighs the risk)

A patient at his end stages of dying. The patient's spouse asks the nurse how she can provide comfort to the patient in his final hours. Which intervention will the nurse advise the spouse to implement? (SATA) A) Getting the patients favorite book and reading it to him B) Help encouraging fluid intake C) Massaging the patient's feet and ankles D) Holding hands E) Having a conversation with the patient

A,C,D,E

A patient is diagnosed with CHF. Which information about self-management at home will the nurse include in the teaching? (SATA) A) Set a regular time for your medication B) Drop down the sodium intake to 4 grams a day. C) Avoid taking ibuprofen for headaches D) Inform symptoms of shortness of breath to your physician E) Report weight gain of more than 3 pounds per week to your physician

A,C,D,E - Adhere to medication regime, avoid NSAIDS {retain salt & fluids}, weight gain and SOB are both exacerbations of CHF ;Pts w CHF need to drop their sodium intake to less than 2 grams per day

A patient diagnosed with asthma is prescribed long-term corticosteroids medication therapy. When the nurse assesses the patient, which manifestations of Cushing syndrome are noticed? (SATA) A) Na++ - 152 mEq/L B) Blood pressure 84/42 mmHg C) High blood sugar D) K+ = 3 mmol/L E) Metabolic alkalosis

A,C,D,E - Cushing syndrome is caused by high concentration of cortisol. Hypokalemia and hypernatremia are common, metabolic alkalosis is also caused by ectopic ACTH production. Hyperglycemia can be caused by long-term corticosteroid use.

Which instructions will the nurse include in the teaching plan about foot care to a newly diagnosed diabetes mellitus? {SATA} A) Dry your feet very well when your shower B) Soak your feet in warm water daily to help with the circulation C) Consult a podiatrist to cut the toenails D) Assess feet every morning E) Always wear footwear while walking

A,C,D,E - These pt have peripheral neuropathy and can not tell if the water is too hot

A 45 year old female patient recently diagnosed with diabetes mellitus. The nurse is educating the patient about signs of hyperglycemia. Which should the nurse include? (SATA) A) Polydipsia B) Sweating C) Polyuria D) Tachycardia E) Polyphagia

A,C,D,E - these are signs of hyperglycemia

The new nurse on the med surg unit makes a medication error. Which actions are appropriate to take in this situation? {SATA} A) Document the dose given in the patient's MAR B) State in progress note the error and that an incident report was initiated C) Assess the patient for any harm from the error D) Inform the attending physician E) Document in the medical record any side effects the patient might have F) Complete an incident report 24-48 hours after the incident

A,C,D,E - {B} an incident report should not be documented, these are only for quality control & dealt with by risk management, {F} start an incident report right away

The nurse provides care for an elderly patient with no past medical history.The nurse evaluates the patient's immunization status. Which of the patient's immunizations are up-to-date? (SATA) A) Receive his tetanus booster at age 60 B) Did not get the hepatitis A vaccine yet C) Last herpes zoster vaccine at age 60 D) Did not get the hepatitis B vaccine yet E) Got flu shot this season

A,C,E - the pt has no medical history {not at risk does not need hep vaccine}

A 33 year old patient is newly diagnosed with a brain tumor. Patient is 12 hours post infratentorial craniotomy to remove his brain tumor. What positions are safe for the patient to be placed in? {SATA} A) Semi - Fowler's position B) Right - lateral position C) Neutral midline head alignment D) Supine position with a soft pillow under the neck E) Slight hip flexion

A,C,E - {B} the question did not indicate a operative side, to avoid displacement you want to place the patient on the non operative side, {D} avoid putting anything under the head that can cause hyperflexion

Patient on perindopril. The nurse determines that further teaching is needed when the patient makes which statements? (SATA) A) I will include more of broccoli and bananas in my diet B) I will be monitoring my blood pressure at least once a week C) I will take my medication every day in morning D) I will use salt substitute with my meals E) I will move slowly from a sitting position to a standing position

A,D - ACE inhibitors can cause potassium retention so a diet that stays away from high potassium is encouraged, salt substitute causes potassium retention

The nurse assesses the new admission. What indicates that the patient is at risk for having a latex allergy? (SATA) A) Patient has a current avocado allergy B) Patient received packed red blood cells C) The patient has a history of arthritis D) Patient has a history of multiple surgical procedures E) Patient is a health care worker

A,D,E - avocados and banana allergies are linked to latex allergy, if a pt has multiple surgical procedures they are exposed to latex a good bit this increases their risk for allergy, a health care worker wears latex gloves constantly, this can increase the risk.

The nurse provides care for a patient diagnosed with neonatal abstinence syndrome. A newborn is having drug withdrawal symptoms. What nursing interventions would the nurse include in the plan of care? (SATA) A) Swaddle the newborn in a flexed position B) Maintain the newborn in a prone position C) Frequently feed the newborn a high-calorie formula D) Station the newborn in a quiet area on the unit E) Dimmed light around newborn's crib provides a more peaceful environment

A,D,E - swaddling the pt will comfort them {similar to when they were in utero} and in a flexed position will prevent startle, place newborn in a quiet, dimmed lighting area to prevent agitation; You do not place an infant prone unattended → SIDS, it is not necessary to frequently feed the newborn

The nurse assesses a patient in labor. Which assessments indicate that the patient is in the active phase of the first stage of labor? (SATA) A) Her contractions are 2 to 3 minutes apart B) Her contractions are irregular C) Her contractions are 30-40 seconds long D) There is an extensive amount of blood gush E) The patient is doubtful of ability to control pain

A,E

The nurse moves a patient from the bed to a chair. Which technique does the nurse use to maintain proper body mechanics?(Select all that apply) A) A wide stance with one foot back and one to the front B )Use upper back muscles when pulling patient to a stand C) Position yourself between the chair and the patient D) Hold patient at arms length when standing E) Pivot toward the chair using leg muscles

A,E

The nurse provides pre-op teaching for a patient. Which kind of anesthesia alters the level of consciousness? (Select all that apply} A) General anesthesia B) Topical anesthesia C) Regional anesthesia D) local anesthesia E) conscious sedation

A,E

A new mother on the maternity unit tells the nurse that she would like to exclusively breastfeed. The nurse provides breastfeeding education to the new mother. Which statements by the mother indicate that further teaching is required? {SATA} A) I don't want my baby to get any bacteria. I'm going to wash my nipple with soap and water after each feeding B) I will massage my breast to help with the engorgements C) I will apply ice to my nipples for comfort D) I will apply lanolin to my dry or cracked nipples E) If my nipples are leaking milk, I will apply pressure and use nursing pads with plastic lining

A,E - these require further teaching; soap and water will cause nipples to become dry and cracked, a plastic lining on leaking nipples will promote moisture and yeast

What is the primary reason for an amputation? A) PVD B) Tumor C) Infection

A- 80% are because of PVD, 75% of those are due to diabetes mellitus

The RN is reinforcing infection control guidelines to a group of nursing students. What data is most important for the nurse to include? A) Always wear a gown when assessing the BP of a patient with a methicillin- resistant Staphylococcus aureus (MRSA) infection B) The door of the patient's room with a C-diff should be closed at all times C) Patient with a hepatitis b infection should eat with disposable utensils D) Surgical mask should be worn when caring for a patient with pulmonary TB

A- Contact precautions are used to control the spread of organisms that are acquired from direct or indirect contact, especially multidrug-resistant organisms such as MRSA

The nurse provides care for a patient receiving external radiation to the abdomen. Which action by the nurse is best? A) Do a thorough assessment and monitoring of the patient's skin B) Monitor white blood count and neutropenia C ) NPO 2 hours before procedure D) Dump the patient's urine in a specific lead-lined containers

A- External radiation is damaging to the skin, must be assessed carefully, B is needed for chemo, c not necessary, d is for internal radiation

The nurse is dropping a urine specimen to the lab. Which nursing action is necessary? A) Always put the urine container in a biohazard ziplock bag B) Deliver by hand to the lab within 1 hour of obtaining the specimen C) Deliver the urine specimen through the tubing system D) Label the urine container with the room number

A- biohazard bags are necessary for all specimens; delivering by hand in 1 hour is not as necessary

The nurse auscultates a patient's bowel sounds. Which finding is most important for the nurse to report to the physician? A) Bruit sound over the abdominal aorta B) Irregular bowel sounds C) Uninterrupted bowel sounds over the ileocecal area D) Absent bowel sounds for a whole 1 minute

A- burit signifies a turbulence ~ Abdominal Aortic Aneurysm {AAA} - emergent; Listen for 3-5 minutes, small bowel sounds are continuous {fluid}, bowel are not "regular"

A patient states "I feel ugly because I lost my hair" after receiving chemotherapy for ovarian cancer. Which statement does the nurse say to help the patient cope with those feelings? A) "Lets see how you look with a shawl or hat" B) "Your hair will grow back 3-4 months after you finish treatment" C) "Some women shave their heads when this starts to happen" D) "Just think how it makes you not brushing your hair every day"

A- the patient has feelings of a distrubed body image and low self esteem because of hair loss. The nurse needs to say something to improve those feelings. Suggesting a scarf or hat is appropriate. It will offer an immediate solution to the patient. C-Telling the pt that shaving her head will not help her cope with her current feelings right away , B dodges the empathy needed with this situation, D- is almost sarcastic

The physician prescribes 1.5 grams of vancomycin to be infused over 24 hours for MRSA infection. How many milligrams per hour does the nurse administer to the patient?

62.5 or 63 mg/hr

12 hours after a cardiac catheterization surgery, the patient has decreased pedal pulses. What is this an indication for? A) Hematoma B) Nerve injury C) Bleeding D) Renal failure

A

The nurse is providing care for several patients that arrived at the convenient care at the same time. Which patient will the nurse assess first? A) 15 year old with a severe headache, stiff neck, and high temp. [meningitis] B) 27 year old with a fever, vomiting, abdominal cramping, and diarrhea. [dehydration] C) 62 year old with arthritis and having severe pain. D) 43 year old with dysuria, severe burning with urination and fever. [UTI]

A

The oncology nurse is giving a teach about risk factors and demographics of lung cancer at a gathering at the local community clinic. Which group of individuals has the highest risk for lung cancer? A)African Americans B) Caucasians C) South East Asians D) Latins

A

What is the therapeutic level for digoxin? A) 0.5-2 B) 0.5-1.5 C) 1-1.5 D) 1.5-2

A

Which of the following is not a sign of severe preeclampsia? A) BP = 152/112 B) Proteinuria +2 C) Oliguria D) Blurred vision

A

A patient diagnosed with liver cirrhosis is admitted to the med-surg unit with ascites. Which treatment will the nurse expect the doctor to order? A) Albumin B) Spironolactone C) Nabumetone D) Fresh frozen plasma

A - Albumin is a hyperosmotic protein solution, it pulls fluid back into the bloodstream

The nurse admits a patient for electroconvulsive therapy scheduled for the afternoon. When collecting the patient history, which finding does the nurse report to the doctor? A) Patient takes alendronate once daily B) Patient states he is depressed C) Patient got ECT treatment 6 years ago D) Patient perform weight bearing exercises 3 times a day to prevent osteoporosis

A - Alendronate is prescribed to treat osteoporosis. ECT contract the muscles and might place the patient at risk for bone fractures

The nurse is taking care of a patient who was sexually assaulted. Which action will the nurse take first? A) Assess and treat any concerning medical problem B) Document traumas to the cervix and perineum C) Call the patient's protected health information (PHI) list D) Help the patient to the bathroom to clean self

A - Assessing and treating medical concerns come first, then other concerns come later

A patient is newly diagnosed with iron deficiency anemia, the nurse is giving discharge dietary instructions. Which meal option indicates that the patient had a good understanding if selected? A) Grounded beef with whole wheat pasta and cheese B) Chicken broth with coffee C) Boiled egg and pickles with orange juice D) White toast with tuna salad

A - Beef is a very high source of iron; Coffee has tannin and will prevent absorption of iron, C has a lot of calcium in it & will help with asportion but there is no iron is this food selection, D is low in iron

A patient presented to the ED with expiratory wheezes and dyspnea. ABGs results are pH - 7.31, PaCO- 50, HCO - 24, and PaO2 - 51. The nurse puts the patient in high fowler's position and administered 1L/min of oxygen via nasal cannula. After 30 minutes, the ABG results are pH - 7.36, PaCP - 44, HCo3 - 22, and PaO2 - 68. Which action does the nurse take first? A) Bump up the oxygen to 2 liters per minute B) Start the process of admitting the patient C) Monitor the patient for another hour D) Discharge the patient home

A - Bumping up the oxygen will improve the hypoxia; normal PaO2 = 80 -100

A patient is being transfused 1 unit of packed RBCs. The patient complains of burning at the IV site. The nurse assesses the site and edema is present. Which action does the nurse take first? A) Stop the transfusion B) Flush the IV cannula with pre-filled normal saline C) Remove the IV cannula and apply pressure to the site D) Raise the extremity above the heart level

A - Burning and edema are signs of infiltration, stop the infusion, assess and raise above heart level; do not apply pressure to the sight,Never flush an IV that has edema

The nurse provides care for a patient with diabetes insipidus. Which nursing diagnosis is most appropriate? A) Fluid volume deficit related to excess urine output. B) Hyponatremia related to high sodium excretion. [not a Nanda nursing diagnoses] C) Risk for fluid volume overload related to decreased urine output [ deficit not overload] D) Hyperglycemia is related to reduced insulin and cortisol production. [diabetes mellitus]

A - DI is deficiency of secretion of antidiuretic hormone or decreased response to ADH. This results in massive water excretion. Thus, fluid volume deficit

Which maternal factors are associated with the risk for fetal macrosomia? {SATA} A) gestational diabetes B) Cigarette smoking during pregnancy C) Pregnancy induced hypertension D) Recurrent UTIs during pregnancy E) Weight loss during pregnancy

A - Gestational diabetes is associated with fetal macrosomia; B causes low birth weight, C is not a risk factor ,D is not a risk factor, E - weight gain is associated with fetal macrosomia

A patient recently diagnosed with non Hodgkin's lymphoma gains 3.5 kg (7.7 lbs) in 2 days. The doctor ordered labs which shows, Na - 126, K - 4.2, Cl - 104, BUN - 18, creatinine - 1.2, Hgb - 10.2, ANC - 2.4K. Which reason does the nurse identify to explain the weight gain? A) SIADH B) Chemotherapy C) Acute kidney failure D) Anemia

A - SIADH this results in increased water absorption & delusional hyponatremia; you wouldn't consider acute kidney failure because BUN and creatinine are normal

The nurse provides care for a patient admitted with right hemiplegia and imparied speech. Which nursing diagnosis is the highest priority for this patient? A) Imparied swallowing B) Self-care deficit C) Risk for injury D) Ineffective coping

A - The patient has difficulty talking, which makes the patient at high risk for aspiration

Alzheimer's patient is currently 48 hours post infection appendectomy. The patient messed with the IV fluid rate and opened the regulator to free flow. When the nurse discovered the incident, the patient already received 500 ml of normal saline. Which assessment indicates fluid volume overload? A) Fine crackles upon auscultation with elevated blood pressure B) Tachycardia and orthostatic hypotension C) Flushed face and warm dry skin D) Thready pulse and increased rate and depth of respirations

A - These symptoms can cause the pt headache and SOB, these are signs of fluid overload

While frying chicken, the hot oil fell on the patient's abdomen and thighs. Patient was diagnosed with 3rd degree burns while in the ED. Which finding will the nurse expect to observe during the emergent phase of the burn injury? A) High hematocrit B) Slow heart rate C) Elevated blood pressure D) Increased urinary output

A - a large fluid shift will occur from intravascular to interstitial

The nurse observes a student assess neonates in the newborn nursery. Which action by the student nurse requires immediate intervention by the nurse? A) Documenting a negative red light reflex in a 2 day old neonate B) Testing the tonic neck reflex by lying the neonate supine and turning the head to one side C) Testing the rooting reflex by stroking the corner of the neonate's mouth D) Documenting a positive Babinski reflex in a 1-day old neonate

A - a negative red light reflex indicates a severe neurological deficit, possibly caused by increased intracranial pressure. It must be evaluated immediately

A 2 year old child with failure to thrive tested positive for a sweat test. What orders does the nurse anticipate? A) Give replacement enzymes B) Apply skin moisturizer C) Restrict salt to 1.5 grams per day D) Start peripheral intravenous hydration

A - a positive sweat test is a positive screening test for cystic fibrosis

A 7 year old boy has been recurrently admitted to the ED for asthmatic attacks. The doctor prescribes Albuterol and Beclomethasone via metered dose inhaler. Which statement by the mother indicates a good understanding of the medication administration instructions? A) I will give my oy Albuterol as needed for shortness of breath, and inhale the Beclomethasone daily in the morning as a preventative measurement B) I will take both inhalers in the morning starting with albuterol C) I will only give beclomethasone when I hear noisy lung sounds D) I will give my boy the albuterol in the morning and the beclomethasone before physical activity

A - albuterol is a rapid-acting bronchodilator used to treat acute asthma attacks. Beclomethasone is an anti inflammatory corticosteroid inhaled form used in the long-term management of asthma

Before a site survey, the nurse manager tells the charge nurse to deny any knowledge of any sentinel events {an unexpected death} if asked by the surveyor. What decision will the charge nurse make? A) Inform the supervisor B) Inform the chief medical director C) Confront the nurse manager and tell her, I'm uncomfortable lying to the surveyor D) Inform the surveyor, the charge nurse was given instructions to not speak to them.

A - always follow the direct chain of command. Nurses have a legal, professional and ethical obligation to tell the truth under any circumstances.; the chief medical director is over the entire facility, start w chain of command; in this situation, confronting the nurse manager will just cause more issues

The nurse provides care for a patient with acute anxiety. What is most important for the nurse to assess the patient for? A) Respiratory alkalosis B) Respiratory acidosis C) Metabolic alkalosis D) Metabolic acidosis

A - anxiety causes hyperventilation, which results in a loss of carbon dioxide. Respiratory alkalosis is caused by decreased carbon dioxide in the blood

The nurse manager is giving orders in terms of patient's room allocation, reviewing plan of care and assigning tasks for LPNs. Which leadership style is she practicing? A)Autocratic leadership B) Democratic leadership C) Laissez-faire leadership D) Transformational leadership

A - autocratic leadership is like a micromanager, this is the aggressive style of leadership, democractic focuses on teamwork, Laissez-faire leadership gives a lot of freedom, transformational leadership will build you up and support the staff

A 6 year old girl with sickle cell anemia is being discharged. Which statement by the mother indicates a need for further teaching? A) I will apply cold compresses to my daughters knees if she complains of knee pain B) I will stay away from crowds especially during flu season C) I Will give my daughter her pain medication every 6 hours when she is in pain D) I will offer my daughter a lot of fluids during the day

A - cold compresses or ice causes vasoconstriction, which block the blood flow even more

A 45 year old patient admitted through the ED with a new diagnosis of CVA.The provider ordered several referrals, which referral is most important for the nurse to work on first? A) Speech pathologist B) Physical therapist C) Skill nursing home D) Clinical psychologist

A - determining if the patient is at risk of aspiration, and nutritional planning is the most important

A home care nurse taking care of a patient receiving tube feeding. Thich care objective will the nurse identify as a priority for this patient? A) Education B) Counseling C) Screening D) Case management

A - education and health promotion are the primary objectives of home care

The nurse provides care for a patient diagnosed with type 2 diabetes. The health care provider has ordered exenatide for the patient. When will the nurse administer this medication? A) Twice a day within 1 hour before morning and evening meals B) Once a day before bedtime C) Twice a day within 2 hours before morning and evening meals D) Twice a day within 1 hour after morning and evening meals

A - exenatide stimulates the pancreas to secrete insulin when blood sugar levels are high. It should be administered twice a day within 1 hour before the morning and evening meals

A 38 year old patient who was admitted for a burning sensation during voiding, turned out to have a UTI. The nurse is getting the admission history. What statement by the patient requires the nurse to dig deeper and ask follow up questions? A) My father had prostate cancer when he was my age B) I am on daily captopril C) Even though I had laminectomy a year ago, I still have back pain D) My desire for sex is at the rock bottom

A - familial history of prostate cancer increases the risk of having it by 50%

A patient diagnosed with AIDS has started to have hyperpigmentation skin lesions on his forearm. What complication is the patient having? A) Kaposi Sarcoma B) Varicella-zoster virus C) Candidiasis D) HSV type 1

A - hyperpigmented skin lesions are associated with Kaposi sarcoma

A 54 year old patient, with a long history of cigarette smoking was diagnosed with laryngeal cancer and had laryngectomy done. The home care nurse instructs the patient's daughter on how to suction the patient's laryngectomy tube. Which observation indicates effective teaching? A) The daughter asks her mother to take several deep breaths before starting the suctioning process B) The daughter uses a Yankauer to suction the patient's laryngectomy tube C) The daughter applies suction while both introducing the catheter to the tube and out of the tube D) The daughter suctions the corners of the mouth from saliva and then the laryngectomy tube

A - hyperventilation prevents anoxia during suctioning

A 15 year old patient fractured his radius bone during gymnastics. During the follow up visit, the patient asks about measures to relieve itching under the cast. Which response by the nurse is accurate? A) Using a blow-dryer, apply cool air under the cast B) Wrap a sterile gauze on a tongue depressor and scratch the itchy area C) Apply ice chips under the cast whenever it itches D) Put hydrocortisone cream on a sterile gauze and apply on the area that itches

A - is very effective

At a community meet up, the nurse is giving education about testicular cancer screening. Which is an accurate statement on testicular self-examination? A) I will examine my testicles on a monthly basis right after I finish my shower B) I will examine my private parts every 3 months C) I will examine each testicle independently by rolling it between my fingers D) I will watch out for any pea sized lump on my testicles

A - it is best to be assessed after a shower because scrotal skin is relaxed and smooth; It should be performed monthly, by examining each testical independently between a thumb & finger, assess for any swelling, or abnormal lumps of any size

The nurse is screening against skin cancer. Which observation most concerns the nurse? A) A firm red nodule with an irregular surface and multiple colors B) A non-palpable pink lesion C) A circular lesion filled with fluids D) A red and shiny lesion

A - looks like a malignant melanoma, {Asymmetry Border Color Diameter Edges}

A 53 year old patient diagnosed with UTI is admitted to the unit for treatment. The patient is on Amikacin every 12 hours and cefepime every 8 hours. Which statement or observation is most important to report to the doctor? A) The TV volume is at the maximum level when the nurse entered the patient's room B) Patient states: "I need to get a new prescription for my eyeglasses" C) I am not liking the hospital food, it tastes like metal D) I think I have a bit of redness on my forearm

A - neurotoxicity manifested by ototoxicity is a serious and irreversible side effect of aminoglycosides

A 40 year old woman at a gynecological visit states that having intercourse is painful. Which action does the nurse take first? A) Ask the patient about her menstrual cycle history B) Order serum hormone levels as per protocol C) Assess in depth the patient sexual experience D) Encourage the patient to perform Kegel exercises

A - patient might be experiencing dyspareunia because of perimenopause or menopause.

The nurse taking care of multiple patients. Which patient will the nurse assess first? A) Patient with CHF and reports retrosternal pain B) Patient who had laparoscopic abdominal surgery 2 hours ago and is sleeping C) Patient who had a left lung lobectomy 15 hours ago and is positioned on the right side D) Patient who had an appendectomy 10 hours ago with vesicular lung sounds on the lower lobes

A - patient w/ CHF reporting signs of MI. Patients with chest pain will be considered in myocardial infarction until proven otherwise.

A 45 year old patient presents to the ED with nausea, headache and diaphoresis. The nurse is obtaining the patient's history and knows that the patient had a T4 spinal cord injury 5 weeks ago after a motorcycle accident. Which action does the nurse take first? A) Get the patient to a sitting position B) Assess the patient's bladder and encourage voiding C) Perform a digital rectal examination D) Administer propranolol as prescribed

A - patient's who experience a spinal cord injury are at risk for autonomic dysreflexia {AD}. If AD occurs first elevate the head of bed,remove restrictive clothing, compression stockings or boots and then check for urinary retention. Give medications if prescribed for HTN. This is a life threatening situation than can occur with spinal injuries about T6

The nurse received an admission from the PACU. The patient had a left total hip replacement. 10 hours later, the patient is still disoriented to place, time, and situation. Which action does the nurse perform first? A) Keep an abductor pillow between the patient's legs B) Keep orienting the patient to the surrounding and situation every hour C) Place the patient on NPO until fully oriented D) Encourage the patient to perform breathing exercises to wash out the CO2 build up

A - placing and keeping a pillow between the legs keeps the new hip joint in proper alignment and prevent accidental dislocation of the prosthesis; with orthopedic surgeries always think safety

A patient with a right tibial wound is prescribed negative- pressure wound therapy. Which is the most important action for the nurse to take prior to applying the wound vac? A) Draw serum protein level B) Assess motor power in the right leg C) Check for capillary refill of the thumb D) Check white blood cell count

A - protein is important for wound healing, if the albumin is low → give albumin to increase wound healing effects

A 47 year old patient has an order of ampicillin 250 mg intramuscular every 12 hours. What information does the nurse use to select the site for the injection? A) The muscle size B) Number of doses needed to be given C) The patient's position in bed D) The size of the needle

A - the best IM site of injection is the vastus lateralis or ventrogluteal muscle, larger muscles

The patient was experinceing hemorrhage 3 hours after her vaginal delivery. The nurse transfused 1 unit of packed RBCs. Which intervention is most important for the nurse to do after the transfusion? A) Check vital signs B) Order hemoglobin & hematocrit C) Assess activity tolerance D) Assess the fluid balance in the last 12 hours

A - the effectiveness of blood therapy is assessed by the hemodynamic stability ;An RN can not order, activity tolerance & fluid balance are not the most important things to assess

A 4-month old infant is scheduled for a wellness check visit. The mother is stating that she tried to introduce blended food to the baby and she stuck her tongue out and spit the food. Which response by the nurse is appropriate? A) That is a normal reflex B) Switch food ingredients, the baby might have disliked the first choice C) Add water to the food to change the texture D) Try pre-packed food from the pharmacy

A - the extrusion reflex of the tongue is a natural reflex for an infant who is not developmentally ready for solid food. This will disappear at 4-6 months of age when the baby is ready to take food.

Which immunoglobulins cross from the mother to the child through the breastmilk? A) IgA B) IgE C) IgD D) Immunoglobulin M

A - the newborn's immature immune system is "boosted" by receiving IgA via breast milk. Therefore, helping to prevent the development of infections or disease

A patient was admitted to the med-surg unit after being exposed to radiation which resulted in a burn. The nurse checks on the patient who looks anxious and complains of abdominal cramps. Which action does the nurse take first? A) Assess the abdominal cramps B) Ask the patient to calm down C) Obtain a physician order of antispasmodic D) Initiate decontamination process

A - the nurse should evaluate the assessment finding and then determine the best course of action

A patient with crutches at the top of the stairs leaning his weight on his good leg and the crutches. What does the patient do next? A) Go ahead to the first step placing down both crutches first B) Bring both the crutches and the unaffected leg down to the first step C) Bring both the crutches and the affected leg down the first step D) Move down the stairs with the good leg first and the crutches follow

A - the patient goes down the stairs with the crutches first

A 64 year old patient with DM type 1 is admitted to a new nursing home but forgets to bring his prescribed medication with him. The patient states that he takes the clear insulin, and the white pill with a heart shape on it. What is the next action by the nurse that is most appropriate? A) I will call your primary care physician to get your medication list B) Do you know what the white pill is for C) Does the medication you take have any side effects D) How long have you been on the white pill

A - the patient's description of his medication is not reliable. The nurse's responsibility to call his doctor and confirm

The night shift nurse is caring for a patient diagnosed with acute kidney failure and is reviewing the patient's flowsheet and notices that the previous nurse didn't record the urine output of the patient. Which action will the nurse take first? A) Call the day shift nurse and ask her about the output B) Initiate an incident safety report C) Ask the patient if he knows his urine output D) Inform the charge nurse about the finding

A - the primary intent is to retrieve the data

A pediatric nurse is taking care of a 6 months old baby who is 6 hours post cleft palate repair. Which intervention by the nurse takes priority? A) Assessing and managing the patient's airway B) Addressing the patient's pain C) Change of saturated dressing D) Feeding the patient

A - the risks for airway compromise is most important

A child had a craniotomy 24 hours ago and just noticed circular clear liquid stains on his pillow case. Which action does the nurse take first? A) Call the doctor B) Write a progressive note with the finding C) Monitor the drainage from the patient's ear D) Collect the drainage in an container for culture

A - this could be a cerebrospinal fluid. CSF is a colorless drainage usually leaks from the nose after brain surgeries

30 minutes after a cystoscopy with spinal anesthesia, the patient starts to feel his toes and move the legs a little bit. Which action will the nurse take next? A) Check vital signs B) Auscultate abdominal sounds C) Assess if legs are warm D) Auscultate lung sounds

A - this indicates the motor blockage from the anesthetic is wearing off. However, patients are still at risk for hypotension. BP and vital signs are important to assess for hypotension

The patient complains of pain at his renal biopsy site 14 hours post op which radiates to his flank and umbilical area. Which complication is suspected? A) Bleeding B) Infection C) Hypertension D) Renal colic

A - this is the number one complication with internal organ biopsy

The triage nurse comes back from her break to find four messages on her patient's to call back list. Which is the priority message to take care of? A) Patient diagnosed with lymphoma is not in the mood for chemotherapy and wants to skip today's treatment. B) A patient inquiring about his abdominal stitches C) A patient with chronic arthritis is complaining of pain and stiff elbows D) Home care nurse stating that the depressed patient is refusing to bathe today

A - this is time sensitive & the patient should be aware of the consequences. Interrupting chemotherapy or postponing a day can change the entire cell cycle

A patient on a psychiatric unit diagnosed with dissociative identity disorder. Which occurrence indicates to the nurse that the patient is improving? A) Patient turns to the nurse when she calls his name B) Patient responds to the nurse's question if he is hungry by "thanks we're fine" C) Patient reports pain in his hip and 10 minutes later reports pain in his leg D) The patient becomes quiet and disciplined after a period of being loud and aggressive

A - when the patient responds to his name, that means that the other personalities are subsided

A 70 year old patient admitted to the med-surg unit. What would the nurse monitor in the patient's fluid and electrolyte balance? A) Hyponatremia B) Hyperkalemia C) decreased oncotic pressure D) Decreased insensible fluid loss

A - with aging ADH increases while renin, and aldosterone decreases, leading to a decreased sodium reabsorption, water retention & hyponatremia; potassium balance is not r/t age. Older patients will have increased oncotic pressure and increased insensible fluid loss

A 12 month old baby is due to receive the first dose of measles, mumps, and rubella vaccine.The nurse realizes that the baby has a low grade fever and a respiratory rate of 36. Which action by the nurse is appropriate? A) Give the vaccine as scheduled B) Delay the vaccine until the baby is afebrile C) Delay the vaccine until the baby is no longer experiencing respiratory systems D) Give acetaminophen and then administer the vaccine

A -The child should receive the vaccine as scheduled. The only contraindication is if the pt is immunocompromised, taking antiviral medication or has MMR allergy

The nurse arrived at their night shift and received a report on four patients. Which assessment finding does the nurse immediately report to the on call doctor? A) A patient with Afib reports halos in his vision B) A patient diagnosed with meningitis complains of headaches C) A patient who is 6 hours post open cholecystectomy, has negative bowel sounds D) Patient reports 2 episodes of diarrhea after he had his CT scan in the afternoon

A -should be reported right away, side effects of Digoxin or Amiodarone are halos or blurred vision; B- headaches are expected finding; C - after 72 hours no bowels sounds would be concerning; D - contrast dye will have laxatives in them

The nurse provides care to a client diagnosed with second- and third-degree burns on the anterior thorax and legs. Which finding will the nurse expect to observe during the emergent phase of the burn injury? A) Elevated hematocrit B) Decreased heart rate C) Increased blood pressure D) Increased urinary output

A) CORRECT— An elevated hematocrit occurs because of hemoconcentration that takes place as the result of the large fluid shifts from intravascular to interstitial spaces during the emergent phase of a burn injury. B) Heart rate increases as a result of fluid shifts. C) Blood pressure decreases as a result of fluid shifts. D) Urinary output decreases because blood is shunted away from the kidneys, and renal perfusion and glomerular filtration is decreased.

The charge nurse is auditing a student nurse performing physical examination of the patient. Which action by the student requires immediate intervention by the charge nurse? A) The nurse asks the patient to say 99 and assess for tactile fremitus B) The nurse opens the patient's mouth and a tongue blade is used to depress the tongue C) The nurse utilizes a stethoscope to palpate the patient's abdomen with the fingers D) After the nurse completed palpating the chest he starts chest inspection

D - physical examination of the chest goes with this sequence: inspection, palpation, percussion, and auscultation

A patient with a new colostomy. Which behavior best indicates that the patient understands how to perform colostomy care? A) Patient correctly explains every step of a colostomy irrigation B) The patient explains to the nurse how to perform the colostomy irrigation while the nurse does the care C) The patient takes a written test about colostomy care and discusses with the nurse the answers D) The patient performs colostomy care while the nurse observes

D - return demonstration by the patient is ultimately the most effective way to assess the patients skills

A nurse is caring for a patient in the ED who reported a sexual and physical assault by her fiance. Which action should the nurse take first? A) Put a consult for a psychiatrist B) Call the chaplain to offer prayer C) Change the patient's wound dressing D) Stay with the patient during the physical exam

D - should provide consistent emotional and physical support especially during the physical exam

The school nurse is watching the kids playing during recess. Which observation requires the nurse to intervene immediately? A) Two boys fighting over a girl B) A boy pushing a girl off the swing while moving C) A 3 year old placing his palm on his ear and crying D) A 6 year old with protruding tongue and drooling

D - signs and symptoms of acute epiglottitis {airway}, stridor, mouth breathing, drooling, hoarse voice, restlessness and cough. Patient feels better leaning forward

Rumors on the med surg unit suspect that the new hired nurse is using cocaine. Which observations are suggestive of that accusation? A) Sneezing & headache B) Diarrhea & nausea C) Fatigue and mydriasis D) Lack of sleep, runny nose, and facial pain

D - signs of inhaled cocaine; {A} are signs of a viral infection or rhinitis, {B} - can be signs of withdrawal from a substance, {C} not r/t cocaine use

A patient with generalized anxiety disorder had been attending therapy programs to help with his anxiety. 3 months into therapy, which statement by the patient indicates to the nurse that progress took place? A) I got rid of my nail biting habit B) When I get overwhelmed with life stuff, I just go to my room C) I get some anxiety episodes every now and then, but they don't last long D) I am getting at least 6 hours of sleep with pleasant dreams

D - sleeping & drawing indicate a well being of the body and a major resolution of anxiety

Which assessment finding indicates to the nurse that the newborn is preterm? A) Presence of lanugo on the upper back B) Cheesy-like gray-white fatty substance on elbows C) Irregular shallow breathing D) Flaccid extremities that are extended when at rest

D - the arms and legs of a full-term newborn are flexed when at rest, this indicates that the newborn is preterm; {A} fine downy hairs (lanugo) are seen in full term newborns, {B} vernix is seen in full term newborns, {C} can be seen in either term

The nurse is assessing a 2 month old newborn for a regular visit, which finding would the nurse consider normal? A) Gibson's murmur B) Head circumference of 42 cm C) Tense anterior posterior fontanel when baby is sleeping D) Extrusion reflex

D - the extrusion reflex causes a baby to push something in his mouth out with his tongue. That instinct tends to diminish around four months of age; gibson's murmur is found in patent ductus arteriosus, the normal head circumference is 33-35 cm at this age, the fontanels should be soft while baby is sleeping

Patient was readmitted 2 weeks after his open cholecystectomy for wound infection. The nurse is to collect a wound culture from the wound discharge. Which step should the nurse implement? A) Give one dose of antibiotics 1 hour before collecting the specimen B) Perform a proper debridement to the wound before collecting the specimen C) Make sure to collect the oozing of the drainage once the old dressing is removed D) Flush the exudate with sterile water before obtaining the specimen

D - the exudate has microorganisms from the wound and the environment. It would not provide an adequate analysis of the organism causing the infection. To ensure that the wound is properly cultured, the exudate should be flushed from the wound prior to taking a swab of the wound bed

While giving medication through NGT. Which action will the nurse take when administering the medication tho the patient? A) Always flush the NGT with 40-60 ml of warm sterile water after administering the medication B) Add the medications directly to the enteral feeding meal C) Check that the ph value of gastric content is less than 7 D) Clamp the NGT for 30 minutes after giving the medications

D - the nurse should clamp the tube for 30 minutes after administering the medication

A student nurse is supervised by the RN while taking BP of an adult patient with a small sized cuff. What instruction does the RN express to the student? A) Reliable readings of BP B) Shows a false low reading C) The high cuff pressure will cause brachial nerve damage D) Shows a false high reading

D - the small cuff will not read brachial artery measurements unless it is extremely inflated, when it is it will show a false high reading

The doctor recommended therapeutic massage for the patient to help with his leg pain. The patient later on asks the nurse to explain therapeutic massage. What would be the best response by the nurse? A) It helps with fluid retention B) It prevents blood clot formation C) Im improves blood pressure readings D) It enhances circulation and muscle tone

D - therapeutic massage helps return blood flow to the heart

The nurse is planning a teaching for a 61 year old patient with a history of recurrent lung infections. Which instruction does the nurse include in the teaching? A) Avoiding cardio exercises B) Don't go outdoors during the winter season C) Limit fluid intake to 500 ml daily D) Don't forget to get your flu shot on a yearly basis

D - this is a preventive method

An alcoholic patient is diagnosed with liver cirrhosis. Which intervention is most helpful for the patient's situation? A) IV albumin B) Kayexalate enemas C) Placement of Blakemore tube D) Low-protein, high carbohydrate diet

D - this reduces the risks of hepatic coma by reducing the level of ammonia that result from the breakdown of proteins; {A} is only used in cases of ascites {B} hyperkalemia is not noted, {C} Blakemore tube is used in cases of esophageal varices and bleeding

After a head trauma, the patient becomes malnourished due to confusion and disorientation. What act indicates to the nurse that the patient is getting better? A) The patient is aware that he has lost some weight B) The patient recognises the different foods on the menu C) The patient states, "I'm thirsty, I want some water please". D) The patient responds to the nurses request when she asks him to eat his food tray

D - this reflects a positive outcome and improvement in nutrient consumption

Patient is going to do a cardiac catheterization. The nurse is performing a pre-op assessment. Which assessment is most critical to perform? A) Height, weight, and BMI B) Check the pedal pulses C) Fluid balance D) Any allergies to iodine

D - very important to assess allergies due to the use of radiopaque dye

The nurse is giving discharge teaching for a patient who had a diverticulitis repair. Which statement by the patient indicates good understanding of the discharge instructions? A) I am allowed to eat fried chicken breast ot french fries once a day B) My breakfast would be four scrambled eggs, half avocado and biscuits with peanut butter C) Smoked salmon with guacamole and fried calamari D) Whole wheat bread with tuna salad & cucumber

D is low in fat and high in fiber

A 10 year old boy was having chest tightness and dizziness. The ECG reveals paroxysmal supraventricular tachycardia. What should the nurse do if the boy experiences the same symptoms at a later stage? A) Ask the patient to sleep on his back with both hand above the head B) Sit in a knee to chest position for 30 seconds C) Perform paced breathing exercises D) Pinch his nose and blow the cheeks

D is valsalva maneuver for vagal stimulation to lower the heart rate

Which task is most appropriate for the nurse to delegate to the NAP? A) Check vital signs during the first 15 minutes of a platelet transfusion B) Assess the patency of the chest tube during the morning care C) Help a newly diagnosed patient with hypertension to fill out the food menu D) Implement bladder training and pelvic exercises for a patient with urinary incontinence

D the nurse should put the plan of care for the bladder training, the NAP can help pt perform the activities. Implementation is within the scope of practice for the NAP, this is not teaching ; Can not assess or teach.

The patient gets his medication via NGT, which medication does the nurse need to call the provider for? A) KCL oral solution B) Phenytoin C) Captopril tablet D) Aspirin- EC

D- Aspirin EC {enteric coated} should not be crushed

A 4 year old boy was presented to the ED with his mother experiencing shortness of breath. Upon lung auscultation, the nurse hears inspiratory wheezes. What is the important question to ask the mother? A) Do you have his immunization record? B) Does the boy have a history of asthma or recent lung infection? C) Did he get his flu shot this season? D) What was the child doing immediately before developing the shortness of breath?

D- based upon the age and size of the child and the symptoms, the pt has an obstruction, swallowed a toy, or aspirated a small piece of food → Inspiratory wheezes ; Asthma has expiratory wheezes

What is the single most important act the preceptor should emphasize on while orienting a new nurse on a burn unit? A) Put on PPE including head and shoe covers B) Make sure medical equipment is not shared between patient rooms C) Put patients with infection in private negative pressure rooms D) Wash hands thoroughly and consistently during your shift

D- hand washing is the single most important act to prevent transmission of infection on a burn unit

What is the most important question for a nurse to ask a 60 year female when obtaining a health history? A) Do you drink coffee B) Check her mother's menopause history C) Check for familial history of osteoporosis D) Are you on calcium

D- older adult females have less calcium absorption; getting the required calcium supplementation is important, prevention is key ; Excessive amounts of caffeine can increase risk for osteoporosis → modifiable; Family history of osteoporosis is non modifiable

A newly diagnosed patient with left breast cancer post mastectomy is given discharge teaching by the nurse. Which statement by the patient indicates proper understanding of the instructions? A) Applying a heating pad under the left shoulder to help with the pain at night B) Apply a sling to your left arm during post op recovery C) Attend the breast cancer support group daily D) Measure the blood pressure with your right arm only D- potentially circulatory impairment or infection could exist due to alterations in lymphatic flow {A} it is not recommended to put a heating pad on or near the operative site, {B} the arm should not be at rest, they should use passive ROM daily, {C} attendance depends on the patient

D- potentially circulatory impairment or infection could exist due to alterations in lymphatic flow {A} it is not recommended to put a heating pad on or near the operative site, {B} the arm should not be at rest, they should use passive ROM daily, {C} attendance depends on the patient

The patient came for her 16 week gestation visit and asked the nurse how her baby looks. Which is an accurate response by the nurse? A) Extremities are just starting to show and you can hear a heartbeat now B) Lungs and ear are functional now C) He would be sucking on his thumb right now D) We can tell the sex of your baby today

D- sex determintion is possible, Extremities are formed & a heart rate is heard by 8 weeks, lungs aren't functional, sucking on his thumb would occur around 20 weeks

The nurse performs a newborn assessment. Which finding does the nurse report to the physician? A) Pink patch on the back of the neck B) Bluish skin over the sacral gluteal area C) A rectal temperature of 98*F D) A respiratory rate of 24 breaths per minute

D: normal resp rate ~ 30-60; Pink patch ~ {stork bite} normal, bluish skin ~ {mangolian spots} normal;, Newborn 0-2yr old, Infants <1 yr old

During an active seizure, is it ok to put a pillow under the patient's head to protect from head trauma? A) True B) False

False - Just let the patient seize, a pillow can cause neck flexion and obstruct the airway

A patient with a left mastectomy should not use her left arm for the first 24 hours to protect the suture line. True False

False - They are encouraged to move the operative side to prevent lymphedema

Azithromycin is a macrolide and should be given with meals to prevent gastric irritation. True False

False - they do not cause any irritation, you give them on an empty stomach

For effective suctioning, advance the catheter 10-15 cm & only suction while withdrawing the catheter. True False

False -Only advance until you feel resistance, then suction on the way out for only 10-15 seconds

In true labor, contractions are stronger, longer & wider apart. True False

False- contractions become stronger, longer & shorter apart

Patient is being treated with a radioactive implant for bladder cancer. The nurse will intervene when the NAP does what? A) Spends more than 30 minutes in the patient's room at a time B) Puts the morning care equipment at the bedside table C) Stands at foot of the bed, crossing the placed portable bedside shield D) Stays at least 3 feet away when speaking with the patient

Neither NAP nor visitors should stand at the foot of bed where the shield is no longer protecting them from radiation

Disseminated Herpes zoster is airborne precaution. A) True B) False

True

Should bisphosphonates be taken on an empty stomach first thing in the morning with 8 oz of water. True False

True -- they should be NPO for 1 hour after too

Patients with internal seed radiation should eat with disposable utensils. True False

True, they should use disposable utensils, remove them from the room in double bags

The patient is scheduled for a Pelvic ultrasound to investigate cervical cancer. What instruction does the nurse include during the pre-procedure teaching? A) NPO for 6 hours prior to the procedure B) The ultrasound transducer might cause some discomfort while examining the abdomen C) Should drink 1 liter of water before the ultrasound D) Don't worry about the pain because local anesthetic will be given

c- Filling the bladder with water prior to the test expands the uterus, causing it to move up and out of the pelvic cavity, which improves visualization

Rifaximin can cause severe constipation in liver cirrhosis patients and might cause bowel obstruction. True False

false -It causes diarrhea

The trigeminal nerve is responsible for chewing. True False

true

Do sickle cell anemia patients have a high risk for CVA? Yes No

yes

Are women with large breasts at risk for breast cancer? Yes No

yes - They are 3x higher than women with smaller breast

The night shift nurse just got a report, she is taking care of a new admission, a 5 year old girl with epiglottitis. Which action is appropriate for the nurse to implement? A) Send the patient to radiology for a CXR with NAP B) Check for a gag reflex using a tongue depressor C) Draw the orders of arterial blood gasses and report the results to the provider D) Assess oxygen saturation SpO2 and initiate the IV antibiotics as ordered

{D} - antibiotics should be started ASAP, epiglottis in school aged children is life threatening; {A} do not send an unstable pt with an unlicensed personnel, {B} never manipulate the airway in epiglottitis → complete obstruction, {C} you don't need ABGs, the pt may also start crying due to the pain → obstruct airway

A patient has autonomic dysreflexia. What is the next step after elevating the head of the bed? A) Give antihypertensives B) Remove restrictive clothing C) Insert urinary catheter D) Call the doctor

B

A patient is admitted with having cutaneous nodular melanoma lesions. A small clear fluid is draining from the lesions. Which personal protective equipment does the nurse use when bathing and changing the linens for this patient? A) Gloves B) Gown & gloves C) Gown, gloves & mask D) Gown and gloves to change the linens; gloves when bathing

B

A patient with CHF is prescribed to infuse 500 ml of NS from 6:00 am until 13:00 in the afternoon. The tubings on the unit have a drop factor of 20 gtt/ml. Which rate per minute will the nurse calibrate the infusion to deliver to the patient? A) 22 gtt/min B) 24 gtt/min C) 48 gtt/min D) 62 gtt/min

B

Paramedics just brought in a patient with facial fractures due to a car accident. What is the most important assessment for the nurse to do? A) Vital signs B) Airway assessment C) Lung sounds D) Wound assessment

B

The patient is admitted from the ED complaining of headache. The patient's BP - 188/102. The patient is known to have a history of hypertension. The nurse should assess the patient's medication adherence. Which statement made by the patient indicates proper medication adherence? A) If my blood pressure is high in the morning, I do take another dose before bedtime B) I usually take my blood pressure medication just after breakfast at the same time every morning. Never missed a dose. C) I monitor my blood pressure every morning, if my blood pressure is normal I skip the dose D) On the weekends, I hike. I take my medication after the hike on the weekends.

B

What is a proper presentation of dementia? A) Sudden onset & reversible B) Slow progressive & irreversible C) Sudden onset & irreversible D) Slow progressive & reversible

B

What is the ideal water irrigation amount before/after NG feeding? A) 10-15 B) 15-30 C) 30-45

B

A 50 years old patient who had a hysterectomy, 48 hours later the patient notices a grayish foul-smelling vaginal discharge. What would the nurse suspect? A) Dehiscence B) Fistula C) Keloid D) Hemorrhage

B - A leakage from the rectum to the vagina is suspected

A patient, who takes medication for hypertension, complaints of a dry cough. Which medication will the nurse suspect is causing the cough? A) Amlodipine B) Lisinopril C) Verapamil D) Propranolol

B - ACE inhibitors most common side effect is a non-productive cough

Can LPN cosign a unit of packed RBCs? A) Yes B) No

B - No,you need two RN's to cosign a blood transfusion

A patient had a myocardial infarction 24 hours ago. The nurse notes that the patient has absent bowel sounds, abdominal distention, nausea, and low appetite. Which complication should the nurse suspect? A) Stress ulcer B) Paralytic ileus C) Large intestinal obstruction D) C. Diff infection

B - Paralytic ileus: obstruction of the intestine due to paralysis of the intestinal muscles. Nausea and low appetite are the initial symptoms.the paralysis does not need to be complete to cause ileus but the intestines muscles must be so inactive that it prevents the passage of food and leads to functional blockage. The MI most likely reduced blood supply and o2 to the intestinal muscles

Paramedics call the charge nurse. They are coming in with 4 patients involved in a motor vehicle accident. Which patient does the nurse plan to see first based on the report? A) A 26 year old screaming of pain. Has a visible right knee deformity and weak lower extremity pulses. [circulation] B) A 22 year old female who is responsive to painful stimuli, pulse rate 118 beats per minute with no visible traumas or injuries. [intracranial hemorrhage] C) A 6 year old boy with a visible deformity to the forearm and crying. D) an 8 year old girl with a cut on the forehead, crying intensely with a pulse rate of 126 beats per minute.

B - Patient might be experiencing internal hemorrhage and go into a hypovolemic shock

The nurse is caring for a patient after a prostatectomy. The nurse is working on a nursing diagnosis of Stress urinary Incontinence related to weakened pelvic musculature. Which goal is most important for this patient? A) Try to retain urine as much as possible B) Minimize urinary incontinence episodes through pelvic exercises C) Offering the patient adaptive clothing D) Get a urodynamic testing to assess the bladder and urethra if they are performing their job of storing and releasing urine.

B - Repetitive exercises are important to strengthen the muscles of the pelvic floor. These might take weeks before seeing any improvement

Which observation by the nurse requires intervention during the care of a patient with continuous NG tube feeding by a student nurse? A) Before refilling the next feeding, the nurse aspirates gastric fluid to check for proper placement B) Student nurse administers two medications at a time, then flushes properly with water C) Maintain the patient in a semi fowler position at all times D) Before refilling the next feeding the student nurse checks and measures the gastric residue

B - Risk for NGT medication clumping and interaction; administer 1 med at a time

The mother of a 3 year old asks the pediatric nurse about suggestions to get her boy to bed without a fuss every night. Which suggestion by the nurse is best? A) Ask your boy, "Are you ready to go to sleep?" B) Tell your boy, "we are going to read your book and then get to sleep" C) Be assertive and tell your boy, "C'mon, it is time to sleep" D) Hand your boy his favorite stuffed toy and walk him to bed

B - The child must learn bedtime limits and have a preparatory transition to bedtime

The charge nurse is rounding on the orthopedic patients on the unit. Which observation of the NAP's actions requires intervention by the nurse? A) the NAP helps turn patient 48 hours post-below the knee amputation onto his abdomen B) The NAP assists a patient wearing a halo vest traction to tie his shoelace C) The NAP raises the patient's arm cast on a pillow D) The NAP is keeping an eye on a patient with hip spica cast who is performing range of motion exercises to the upper extremities

B - The patient should not bend because the weight of the halo vest is likely to cause humto fall forward; the NAP should tie the pt shoes not "assist" him

A patient is on peritoneal dialysis. The nurse notices that the outflow looks tinged red. Which action does the nurse take first? A) Call the doctor B) Ask the patient about her last menstrual period C) Check vital signs D) This finding is normal in peritoneal dialysis

B - because of the hypertonicity of dialysate, blood from the uterus can be pulled through the fallopian tubes into the effluent; It should be clear in color, or tinged yellow

What is the most important assessment for a patient post thyroidectomy? A) Flaccid muscles B) Numbness and tingling in the fingers and toes C) Leg pain D) Confusion

B - common complication during thyroidectomy is removing the parathyroid glands which would cause hypocalcemia. Numbness and tingling around the mouth, fingers and toes along with muscular twitching are signs of hypocalcemia. Flaccid muscles indicate hypercalcemia, confusion results from hyperthyroidism

The spouse of a patient with depression is stating that the patient doesn't complete activities of daily living. What is an appropriate intervention for the nurse to suggest? A) Give the patient his medication before planning activity B) Allow the patient extra time to finish scheduled activities C) Help the patient with dressing and grooming D) Force the patient to complete morning ADLs

B - communicating clear, calm expectations and giving the patient time to complete activities are useful and will benefit the patient to complete his activities; Depression medication take 2-4 weeks to give a therapeutic effect, some medication can also cause drowsiness

A 55 year old patient was ordered dexamethasone 4 mg BID for nausea and vomiting. Which patient statement causes the nurse to be concerned? A) I will take my medication with meals B) I have this wound on my toes that won't heal C) I have to get consistent workouts D) I will avoid crowds from september till march

B - corticosteroids suppresses the immune system → this will affect the healing process, can cause infection

The nurse is teaching a patient about hemodialysis. What indicates that the teaching has been effective? A) I will be having weekly dialysis B) Muscle cramping is a common side effect of dialysis C) I will need to take anticoagulants because dialysis slows down my clotting time D) Dialysis will lessen the risk of getting infections

B - due to the rapid removal of body wastes, fluids, and electrolytes, teh muscles might cramp; Dialysate is done 2x a week ,With dialysis you have an increased risk for infection

The night shift psychiatric nurse on the unit is going out of the patient's room and notices a bipolar patient experiencing an episode of mania at the nurse's station. Which intervention would promote therapeutic interaction with the patient? A) Approaching the patient and assertively ask him to abide by acceptable behaviors B) Redirect the patient to his room and minimize trigger of his behavior C) Reminding the patient of the limits and isolating the patient to his room D) Going over the unit rules and the patient's rights, then encourage the patient to go to his room and sleep because it is late

B - effective strategies include redirecting the patient and reducing environmental trigger

A 44 year old patient presented to the ED complaining of chest pain and shortness of breath. Vital signs as follows, BP - 116/67, and irregular HR, and RR of 30. Which order will the nurse implement first? A) Obtain a 12 lead EKG B) Administer oxygen C) Subcutaneous morphine sulfate D) Fluid restriction to 500 ml/24 hours

B - ensure proper oxygenation to the cardiac muscles

When is Nortriptyline given? A) A.M. B) P.M.

B - given at night to minimise the sedative effect

The psychiatric prison nurse is at a community meet-up at a rehabilitation center talking to ex-convicts about different stressors after getting out of prison. A survivor of innmate partner violence is having anxiety issues and was asking about the appropriate therapy for his situation. What is the nurse's best response? A) Mindfulness-based cognitive therapy B) Group therapy C) SSRI medication D) Behavior modification

B - group therapy is best for those type of patients; {A} is used for depressed pts, {C} are used for psychotic pts; {D} can be used to adjust the behaviors of pts with anxiety but it is not the best option

Which action is most important for a nurse to take with a patient getting radiation through implanted seeds for his bladder cancer? A) Check the position of the seeds every 4 hours B) Put the patient on a low-residue diet to minimize bowel movement C) Instruct the patient to use the bathroom every couple of hours D) Minimize drinking fluids to increase the effect of radiation in the bladder

B - having a bowel movement can dislodge the radioactive implant, pt may be put on stool softener; Check the seeds q 8 hours, the pt needs to stay on strict bedrest, increase fluids to wash the radiation out of the body

The nurse double dosed the patient on vancomycin and the patient is at the oliguric phase of acute kidney injury. Which will the nurse expect to observe? A) Urine output of 1ml/kg/hr B) K + 5.9 C) Creatinine 1.1 D) BUN 16

B - hyperkalemia is expected in AKI ; for AKI you would see < 0.5 ml/kg/hr not 1ml; the creatinine normal 0.5 - 1.2; BUN is normal 10 -20

The nurse witnessed a visitor collapse in the patient's room during her day shift. What is the most important action for the nurse to take? A) Start chest compressions B) Shout for help C) Tap the visitor on the shoulder and shout, "are you alright?" D) Check for carotid pulse

B - if someone is witnessed a sudden collapse, thich is more likely to be related to a sudden pulseless arrhythmia, the lone responder is advised to call for help then perform CPR

After being in a long term care facility for a month due to rehabilitation from a back injury. A 62 year old patient asks the nurse about exercises to improve and maintain optimal musculoskeletal function. Which response by the nurse is best? A) Do not perform weight bearing exercises B) Perform resistance training C) Putting on muscle in older adults is an unrealistic goal D) Older adults should stick to walking as a form of exercise to maintain activity

B - it helps maintain optimal function in older adults

A sickle cell patient is admitted to the hematology oncology unit for a blood transfusion. Which fluid is the solution of choice to go with packed red blood cell transfusion? A) LR B) 0.9 % NS C) D5% in 45% NS D) 0.45% NS

B - it is isotonic and does not mess with the hemostasis of the blood

The nurse is taking care of a patient with third degree burns. While planning for the dressing change of the burn, the nurse gives priority to which action? A) Get all the necessary medical supplies and bring them to bedside B) Schedule a time to change the dressing C) Inform the patient about the dressing and prepare him mentally for the pain D) No visitors allowed during the dressing change

B - it is very important to schedule the dressing into the plan of care so that the patient can be mentally prepared ad the nurse will plan the time to administer the pain killer before the dressing change

The nurse provides care to a new admission. Which action is the best example of culturally appropriate nursing intervention? A) Assign the family members to most of the patients personal care B) Take a permission before touching the patient for the physical assessment C) Keep a personal space of 3 feet when assessing the patient D) Keep in mind the patient's ethnicity as the most important factor in planning care

B - many cultures consider it disrespectful to touch a patient without asking permission; in a NEW admission, the ethnicity is not the most important factor

The nurse wants to determine the heart rate while holding a 6 second strip. The nurse counts 7 QRS complexes. What is the heart rate? A) 88 bpm B) 70 bpm C) 80 bpm D) 32 bpm

B - on a 6 sec strip count the QRS complexes x 10

Paramedics presented to the ED a patient with a flail chest on the right side after a motor vehicle accident. Which finding will the nurse expect during her assessment? A) Chest on the right side moves outwards on inhalation and pulls inward on exhalation B) Chest on the right side pulls inward on inhalation and moves outward on exhaling C) Inspiratory wheezes on right side D) Absent movement on the right side

B - paradoxical respiration is when the chest pulls inward during inhalation and moves outward during exhalation → opposite of normal respiration {notice the two completely opposite choices A/B. The answer will always be one of them}

The nurse cares for a patient with an eating disorder. The nurse knows that it is most important to assess for which problem? A) Aggressive behavior B) Poor self-esteem and feelings of altered body image C) Hard time accepting reality D) Family acceptance issues

B - patient's ability to be an advocate for self is difficult to accomplish

The Night shift nurse gave a report to the coming nurse of a patient who is on his way from the ED. The patient has a history of COPD who presented to the ED complaining of dyspnea. What does the nurse expect to observe? A) Low blood pressure B) Shortness of breath C) Productive cough D) High heart rate

B - patients who experience dyspnea will state that they feel "short of breath" or are unable to "catch their breath"

The night shift psychiatric nurse receives reports on four patients. Which patient does the nurse see first? A) A patient going through divorce and depression, after she gave birth to a baby with growth retardation B) An ex convict with a history of post traumatic stress disorder {PTSD} after having physical abuse in prison admitted with agoraphobia after someone called the cops on him C) A business owner who is diagnosed with severe anxiety because he shut down his business and filed bankruptcy D) A dancer admitted for depression and anxiety after her sister committed suicde and her mother was recently diagnosed with ovarian cancer

B - patients with agoraphobia have a tendency for violence to self and/or others

The nurse on the dialysis unit is ready to discharge the patient home on continuous ambulatory peritoneal dialysis. Which statement by the patient indicates proper understanding of the instructions? A) I will drink distilled water every 2 hours B) I will cover the catheter during dwell time C) I will warm the dialysate before using it D) I will disinfect the AV fistula with chlorhexidine BID

B - peritonitis is a serious complication if aseptic technique is not used when caring for catheter; {A} is not necessary, {C} do not warm, open pre wrapped package and use sterile technique, {D} AV fistula is only used in hemodialysis

A patient diagnosed with peripheral arterial insufficiency is ready for discharge. It is important to include which of the following in the discharge teaching? A) Soak your feet in warm water three times a day B) Keep your body warm by wearing baggy layered clothing or use blankets C) Elevate your feet whenever you are resting D) Lay down your feet on the floor for at least 4 hours a day

B - stay warm by dressing in non-constricting layers of clothing; resting your feet on the ground is recommended but 4 hours a day is extensive

The night shift nurse received a report from the ED of a new admission complaining of shortness of breath. Upon admission to the unit, the nurse assesses the lung sounds of the patient. Which type of assessment is the nurse completing? A) Body systems B) Focused C) Comprehensive D) psychosocial assessment

B - the assessment performed based on a disease process or symptom is a "focused" assessment

A medical surgical nurse calls in sick 2 hours before the beginning of the shift. The supervisor floats a nurse from the postpartum unit. Now the team consists of a nurse, a NAP, and one nurse resigned from the postpartum unit. Which patient will the charge nurse assign to the postpartum unit nurse? A) Patient with lumbar spinal cord injury required assistance to transfer to the bedside commode B) A patient with a history of MI 2 days ago who is having insomnia C) A hospice patient exhibiting gasping respirations D) A 24 hour post-craniotomy patient who is on continuous sedation

B - the float nurse will be assigned to a stable patient

The gynecology nurse navigator is initiating a plan of care for the patient. The nurse is preparing the patient for a pelvic exam. During medication reconciliation, it appears that the patient is on antihypertensive medication. Which statement by the patient requires the nurse to do further assessment? A) My last pap smear was 4 years ago B) Dong Quai helps my hot flashes C) I do yoga daily for 90 minutes D) My partner doesn't like using condoms

B - the nurse assesses the patient's needs and educates the patient regarding possible hypotension

The patient was presented to the ED with drowsiness and cold clammy skin. The finger stick blood sugar shows 36. What would the nurse's priority intervention be? A) Check serum glucose level B) Give IM glucagon C) Give the patient a full glass of orange juice D) Get an urgent ECG

B - the nurse must recognize assessment finding that indicate hypoglycemia dn be prepared to implement preferred treatment; if the patient has an altered mental status {drowsiness} you do not want to give the any oral fluids

A patient is at 11 weeks gestation. Which finding does the nurse expect to observe? A) Can feel the fundus at the umbilicus B) Can listen to the fetal heart rate using a doppler C) Can palpate the fetal movement D) Can see colostrum

B - the nurse should be able to hear the fetal heart rate of 110-160 bpm with doppler around 8-12 weeks gestation; at this time the fundus will be at the symphysis pubis, fetal movement is felt around 18-20 weeks, colostrum is seen at 16 weeks

During a patient education seminar about colorectal cancer. Which patient does the nurse identify as being at higher risk for colorectal cancer? A) A 20 year old patient who exercises regularly B) A 56 year old patient who follows a high fat diet C) A 32 year- old adult who has an uncle with colon cancer D) A 45 year old female who had hysterectomy 2 years ago

B - the patient has a couple of risk factors, age >50 years old, and following a high fat diet; familial history of 1st degree relatives {parents, siblings} is a risk factor not an uncle.

During the first home visit, the nurse notices severe epistaxis. The patient was discharged after a myocardial infarction. It is most crucial to ask which question? A) Do you have any allergies? B) Do you take an anticoagulant medication? C) Did you pick your nose today? D) Are you on any NSAIDs medications

B - the patient might be taking anticoagulation for his cardiac problems. Epistaxis Is a side effect of anticoagulant medications.

The nurse delegates ambulating a patient 4 days after surgery to a nursing assistive personnel (NAP). Which type of delegation is the nurse following? A) Supervision B) Circumstance C) Person D) Communication

B - the person is stable 4 days post op

A congressman was admitted to the med-surg unit for investigation to rule out colon cancer. The doctor notices while rounding that the nurse taking care of the patient is messy and unkempt and asks the charge nurse to reassign the congressman to another nurse. What would be the best response by the charge nurse? A) I can talk to the nurse to go to the restroom and refresh her appearance B) With all the respect, I can not respond to your request C) Do you have a preference of who to assign to the congressman D) I didn't notice that she is that messy

B - the request to reassign a nurse based upon physical appearance is unacceptable. Assignments are based on skillset and competencies

The patient lost his left leg due to a shark attack. The patient is 72 hours post above the knee amputation. The patient expressed interest in prosthesis. Which action should the nurse take when caring for this patient? A) Motivate the patient to sit in the chair for at least 10 hours a day B) Keep the antiembolic stockings to the left leg C) Encourage range of motion exercises in the morning D) Raise the left leg above the heart level for 3-5 days

B - this helps shaping and molding for eventual prosthesis fitting. A compression dressing is typically ordered to reduce edema, support soft tissues, and promote limb shrinkage and maturation ;You don't want ATK amputations to sit in a chair for long periods of time, range of motion exercises should be done more than once a day, leg should be flat to prevent hip contracture

The nurse in a rehabilitation center is taking care of a patient diagnosed with alcohol abuse. HWich statement by the patient indicates that he is using projection as a defense mechanism? A) Alcoholism is a genetic predisposition B) My wife lives on antipsychotic drugs 24/1, I don't live that way C) I only enjoy 1-2 glasses of red wine while reading t night D) I have seen a lot of youtube videos that say alcoholism is not a disease

B - this involves attributing their own unacceptable thoughts, feelings, and motive to another person; {A} is rationalization, {C} is denial, {D} is intellectualization

A 30 year old patient was diagnosed with stage III lung cancer. Two weeks later the patient tells the nurse during his clinic visit that he feels great and thinks the doctor made the wrong diagnosis or his tissue sample was mixed up with another patient's sample. Based on the stages described by Kubler-Ross the patient is experiencing which stage? A) Acceptance B) Denial C) Anger D) Bargaining

B - this is the first stage of the grieving process, pt may also request a second opinion

A COPD patient diagnosed with emphysema. The patient starts getting anxious and confused. What is the first action the nurse should take? A) Raise the oxygen flow rate from 2 to 4 liters per minute B) Help patient perform breathing exercises C) Check the patient's serum sodium D) Assess the patient's blood pressure

B - this prevents the collapse of the alveoli and helps the patient control the depth and rate of breathing

How will the nurse put the trochanter roll into position for an unconscious patient? A) Trochanter to the knee B) Lateral side of the hip to the middle of the thigh C) Middle of the thigh to the knee D) Medial part of the hip to the mid part of the calf

B - this prevents the legs from rolling outwards when the patient is in supine position. If patella is directed upwards, this means that the position of the roll is correct and effective.

Nurse is triaging patients. Which patient has priority to be seen first? A) A 20 year old who got out of a car accident with neck pain B) A 4 year old having hoarseness and noisy, high pitched wheezes on inspiration C) A 9 year old who got his scheduled immunization yesterday is having a fever of 101.5F D) A 45 year old patient on coumadin who fell down and has deep laceration of his tibia

B - those signs are concerning of stridor which indicates an upper airway obstruction which is an emergency, airway is the highest priority

Which of the following is not a sign of the dirtectic phase of ARF? A) Hypovolemia B) Hyperkalemia C) Hyponatremia D) Low USG

B - you will have hypokalemia in the diuretic phase

The RN is reviewing how to prevent medication error with a nursing student. Which response by the nursing student indicates that additional teaching is required? (Select all that apply) A) Preparing medications for patients independently B) Checking the MAR against the drug level at least two times prior to administering the medication C) Trusting the health care provider and not questioning a medication ordered D) Instant documentation of all medications when given E) Using two patient identifiers when administering medications

B,C -Check the MAR at least 3x {pulling drug, preparing medication, at bedside}, verify orders

The nurse provides care for the patient diagnosed with esophageal cancer. Which goal does the nurse establish in the plan of care? (SATA) A) Patient will be in complete remission of his cancer B) Patient will be able to eat without aspirating C) Patient's pain will be controlled D) Patient will maintain his baseline weight E) patient will consider hospice care

B,C,D - these are appropriate goals for the nursing plan of care

A medical surgical nurse admitted a newly diagnosed patient with DVT. The provider prescribed warfarin 20 mg BID for the first 15 days of therapy. While collecting the initial assessment history, which statement would warrant the nurse to hold warfarin and contact the provider immediately? {SATA} A) Patient had a bilateral cataract 2 weeks ago B) Recent endoscopy revealed gastric ulcer C) Patient is a daily alcohol consumer D) Patient is on a baby aspirin daily for the past 5 years E) Recently colonoscopy revealed multiple polyps

B,C,D - warfarin have an increased risk for bleeding and are contraindicated w alcohol & aspirin

A patient with renal failure was admitted to the ED with severe pyelonephritis. During the oliguric phase, what does the nurse expect to see? {SATA} A) Urine specific gravity of 1.041 B) Itching C) Azotemia D) K+ 5.9 E) Vomiting

B,C,D,E

The nurse instructs parents on the sudden infant death syndrome (SIDS). Which statements require the nurse to act? (SATA) A) My baby sleeps in a supine position B) My baby sleeps in his car seat on my way to child care C) I always cover my baby from chest to toe during sleep D) I like to put a small pillow under my baby's head when he sleeps E) I let my baby nap on my waterbed

B,C,D,E - An infant has no control over their neck muscles, if they fall asleep in the car seat with their head leading forward it could obstruct or close their airway, A blanket and pillow could cover their face and cause suffocation, an infant should sleep on a firm surface, soft surfaces could cause neck flexion or suffocation ;Supine is an appropriate position to sleep in

When the nurse enters the patient's room, she notices that the patient who is on continuous NGT feeding has a weak cough and dusky lips. Which actions should the nurse take? {SATA} A) Keep the feeding as ordered B) Assess lung sounds C) Hold the tube feeding D) Call the doctor E) Suction the patient's mouth

B,C,D,E {suction the pts mouth to keep a clear airway}

The patient is broke. The patient calls the nurse to ask if he can substitute the prescribed Pantoprazole with an over the counter antacid with a cheaper price. Which response by the nurse is appropriate? {SATA} A) Let us try the antacids for a week and will give you a follow up call then to see how you feel. B) Let me check with the pharmacy about the cost C) Pantoprozole is a selective "proton pump inhibitor", a medicine which reduces the amount of acid produced in your stomach. It is used for treating acid-related diseases of the stomach and intestine D) Caffeinated beverages can also help in the ulcer healing process E) I will email educational videos about smoking cessation F) Call the clinic if you experience black tarry stools

B,C,E,F

Which statement if said by a patient receiving doxycycline raises concerns? {SATA} A) I should wear a sunscreen during my morning walk B) I have noticed a greenish vaginal discharge C) I will take my doxycycline pill at 11 am and 11 pm D) My boyfriend uses condoms for birth control E) I will complete taking all the pills prescribed to me F) I will take my antacid at bedtime

B,C,F - B - dox. Has a risk to increase superinfection, C - it is taken at least 1 hour before because it causes esophageal irritation, F antacids should be taken 1-3 hours from dox administration, A - Doxycycline causes photosensitivity, sunscreen is suggested. D - it interferes with oral contraceptives, E correct information

The charge nurse is rebooting mid-day and is coordinating care for patients on a med-surg unit. What activities can the nurse delegate to the NAP? {SATA} A) Assign room placement for patients coming from the ED B) Deliver a patient to the radiology unit for an abdominal ultrasound C) Remove a patient's peripheral IV D) Assist the doctor with a patient's vaginal exam E) Prepare the equipment needed for a peripheral IV insertion F) Explain to the patient that his chest pain needs to be assessed immediately in the ED

B,D

A healthy patient is being treated for localized herpes zoster (shingles). What observations indicate to the nurse that care is appropriate? (SATA) A) Applied airborne precautions B) Prescribed oral acyclovir C) Keep patient's room closed at all times D) Nurse assigned to care for patient has history of chickenpox E) Perform pursed lip breathing exercises

B,D,E - Herpes zoster (shingles) is contact precautions, Acyclovir should be prescribed, nurse w a hx of chickenpox is immune to virus, pursed lip breathing can help the pt cope with the discomfort of shingles {does not affect oxygenation or respiratory system}

The nurse is taking care of a 60 year old patient with poor eyesight. What are the best interventions to meet the patients needs? {SATA} A) Maintain a monotonous voice B) Describe the sounds in the surrounding C) Minimize surrounding noise before speaking with the patient D) Stay within the patient's visual field E) Introduce yourself by name and title once you speak with the patient

B,D,E - if there are noises that are coming from an area that the pt cannot see, you should explain what is going on, stay within the patient's visual field, introduce yourself so that the pt does not become confused and can potentially recognise you and your intentions in the pts care

A patient with B-cell leukemia is getting high dose methotrexate. Which statement by the patient shows understanding of appropriate precautions? (SATA) A) Brushing and flossing my teeth twice a day is very important B) Visitors to my room should not bring flowers C) I will bring my own pillow and blanket from home because I can't sleep without them D) I will only shave with my personal electric razor E) I will only allow visitors who have a cold into my room if they wear a mask

B,D- Methotrexate is a chemotherapy drug. Patients receiving chemo are recommended to use soft toothbrushes or sponges, never floss {high tendencies to bleeding}; flowers are a high medium for bacterial growth, pt are at high risk of infection

Which activity does the nurse delegate to nursing assistive personnel? (SATA) A) Teach the patient how to self-administer a small enema B) Take vital signs of a patient with contact precautions C) Take the patient's vital signs at 10 minutes from the blood transfusion start D) Reinforce the dressing of a pressure ulcer E) Help a patient with poor eyesight to get to the bathroom

B,E

The charge nurse is planning an assignment for the night shift team. The team consists of an RN, an LPN, and a NAP. Which patient does the nurse assign to LPN? {SATA} A) A newly diagnosed patient with HSV B) Patient diagnosed with C. Diff infection C) A patient admitted with pneumonia on the day shift D) Patient post cardiac catheterization who is on IV heparin E) A patient with Parkinson disease admitted for fatigue

B,E - these are stable patients with predictable outcome; {A} since they are newly diagnosed they may require education and may be unstable, {C} the pt was admitted less than 24 hours ago, their stability and outcomes are unknown, {D} this pt does not have predictable outcomes

What is a straight violation to the EMTALA {Emergency Medical Treatment and Active Labor Act}? {SATA} A) Failure to provide advanced directive information B) Patient is denied for medical screening after few visit to the ER complaining of the same flank pain which doctors couldn't find a diagnosis to in the previous assessments C) A nurse shares the patient's medical health information with a family member who is not on his PHI {patient health information} access list D) A patient is selling his kidney after his business failed E) Transferring a patient to another facility before stabilizing his condition

B,E ; {C} violates HIPAA, {D} violates the NOTO

Which statement by a patient diagnosed with CHF raises concerns for the nurse? A) When I wake up my ankles are normal, but are swollen at night B) I have hard time catching my breath after I make my bed in the morning C)I feel bloated most of the time D) Every time I cough, I experience chest pain

B- Pulmonary edema is more concerning fluid passes from the pulmonary capillaries to the alveoli causing SOB, and respiratory depression, dyspnea can become worse with physical exertion; Chest pain caused by coughing is a sign of pericarditis and can be treated with antibiotics, Swollen ankles and the feeling of bloatedness are expected signs for right sided HF

Which tasks can be delegated to a UAP? A) Inserting a foley catheter B) Measuring and recording the patient's output through a Foley catheter C) Teaching a patient how to care for a catheter after discharge D) Assessing for symptoms of a urinary tract infection?

B- falls w/i the implementation phase of the nursing process and does not require nursing judgement. Evaluation of the intake & output must be done by the nurse

A new NAP is on the unit, the nurse delegates finger-stick glucose check and then discovers that the NAP has never done this task before, and changes the assignment. Why did the nurse change the assignment? A) It is not the right supervision B) It's not the right person C) It's not the right circumstance D) It's not the right direction

B- it is not the right person. Choosing the correct personnel to complete the task {nerve done it before}; Supervision {appropriate monitoring, intervention and follow up}, circumstance{using appropriate pt & setting to determine if it is right}, direction{giving the right direction for a task}

The patient calls the nurse and complains about pain at the IV insertion site. He has dobutamine infused via peripheral intravenous catheter. After stopping the infusion, which action does the nurse take next? A) Apply cold compresses to the IV site B) Call the doctor C) Mark around the affected skin D) Position and support the extremity

B- signs of extravasation should be reported right away to start treatment before losing any tissue, Never apply cold compresses to dobutamine; marking the area and supporting the extremity comes after treatment is initiated

The nurse assesses a 7-month old baby during a wellness check visit. Which finding would the nurse expect on this assessment? A) Has a pincer grasp B) Sit up without support C) 3x the baby's birth weight D) Palpable posterior fontanel

B- sit up without support is a realistic expectation; Pincer grasp appears at 9 months, 3x birth weight by the age of one, posterior fontanel closes at 2 months

a 52 year old patient had a fever and sore throat for three days. After a throat culture, the patient was diagnosed with a strep throat. Urinalysis shows protein 2+ and moderate RBCs. The patient was started on amoxicillin 875 mg for 10 days. Patient stated that he is leaving on a mini vacation in 5 days for 3 weeks. What would the nurse recommend? A) Check for penicillin allergy B) Schedule the patient for an office visit before his vacation C) Where is he going on vacation D) Ask the patient to present his airfare ticket

B- the pt is experiencing a strep throat. One adverse effect of this bacterial infection is acute glomerulonephritis, which affects the kidney's ability to function appropriately. Patient should be seen before and after his vacation

Which of the following electrolyte imbalances can cause seizures? A) Hypernatremia B) Hyponatremia C) Both

Both can cause seizures - values that are not within normal range of 135-145

What is the most common complication after joint replacement surgery? A) Clots B) Nerve damage C) Infection D) Bleeding

C

What is the treatment of unstable V-tach with a pulse? A) Defibrillation 200 J B) Amiodarone C) Sync. Cardioversion D) Cardiac Compressions

C

When do you administer the Herpes live vaccine? A) 40-50 years B) 50-60 years C) > 60 years

C

Which manifestation of DVT needs to be addressed immediately? A) Leg pain B) Leg edema C) Tachypnea D) Erythema

C

Which organism causes epiglottis? A) MRSA B) Staphylococcus aureus C) Haemophilus influenzae D) Streptococcus pneumonia

C

An asthma patient suddenly develops expiratory wheezes. Which class of medications does the nurse give first? A) Theophylline B) Prednisone C) Albuterol D) Diphenhydramine

C - Albuterol is a rescue medication. B2-adrenergic agonists are the most effective to help with acute bronchospasm. ;A → theophylline takes 1-2 days to start working, B) prenidosen takes 1-2 weeks to have a therapeutic effect, D→ antihistamine

The patient is diagnosed with sleep apnea and requires a CPAP at night. While relaying the discharge teaching, which statement by the patient indicates that additional teaching is needed? A) The mask will cover my nose and mouth B) The air is pressured to my upper airway to keep it open C) I will step out to the balcony to have my smoke D) My CPAP will be my companion during vacations

C - CPAP uses room air not oxygen, and is not combustible

A patient with Burkitt's lymphoma is diagnosed with spinal cord compression and is being treated for a neurogenic bladder. The patient was started on chemotherapy ang the compression was resolved. The plan is to retrain the bladder. Which medication does the nurse expect to be ordered? A) Diphenhydramine B) Lorazepam C) Dicyclomine D) Bethanechol

C - Dicyclomine is an anticholinergic medication that promotes urinary retention; {A} is an antihistamine, {B} is an antianxiety med, {D} has a cholinergic effect that promotes urination

The nurse receives a report front eh night shift. Which patient does that nurse see first? A) A patient 3 days post CABG is getting his atrioventricular wires removed today B) Patient with diabetes mellitus type II is getting a cardiac catheterization at 12 noon C) A patient at 36 hours post surgery with a PCA for pain D) A patient awaiting heart transplant

C - PCA is used for pain relief but can have side effects from the narcotics. The nurse should monitor hypotension, respiratory depression, nausea, bowel movement & urinary incontinence

The patient is having status epilepticus. Which action is most appropriate for the nurse to take? A) Place a mouthpiece in the patient's mouth B) Hold the patient's arms down C) Put the side rails up D) Maintain the patient in the center of the bed

C - Putting up the side rails

Which statement by a patient diagnosed with systemic lupus erythematosus raises concerns for the nurse? A) My energy fluctuates from day to day B) I've got a vacation coming in 2 weeks C) Now that the spring is here I am planning to do more garden work activities D) My face rash is fading, that makes me more comfortable in public

C - Sun exposure with trigger exacerbations, micro abrasions can become infected, joint pain are major concerns that need follow up.

A patient is presented to the ED with loss of consciousness. The patient is diagnosed with DKA. The ABGs results are pH - 7.22, PaCO -37, HCO - 19, and SpO2 - 88. Which order by the doctor requires questioning? A) Oxygen 2 liters/ minute via nasal cannula B) Glucose finger stick check Q 1 hour C) Give sodium bicarbonate 100 mEq over 2 hours D) Repeat ABG's in 1 hour

C - The current algorithm in the management of DKA recommends giving sodium bicarbonate if the pH is below 6.9

A mother of a 2 month old infant is inquiring about birth control while breastfeeding. Which statement by the mother indicates a follow up teaching is required? A) I am planning to get a diaphragm installed B) My husband needs to start wearing condoms C) My husband doesn't like condoms, so I will start using oral contraceptives D) I will get an ovulation test and not have a sexual contact when the test is positive

C - The hormones within the contraceptive medication adversely effects breast milk production

During a wellness child visit of an 8 month old baby. The nurse is to perform a physical assessment on the infant. What is a good technique for the nurse to use? A) Don't wake the infant to auscultate the heart sounds B) Fully undress the infant to do the physical assessment C) Ask the mother to hold the infant during the physical exam D) Start the physical with assessing the nose and throat

C - The infant may be fearful of the nurse and equipment, the mother will bring the infant comfort ; {A} you want the infant sleeping {B} leave the infant dressed, undress to assess certain areas toward the end of the assessment {D} start with the least invasive assessments to last

The radiation oncology nurse is educating a newly diagnosed patient with cervical cancer about insertion of radiation seeds. Which statement by the patient indicates a need for further teaching? A) I will not be needing to have a bowel movement during treatment B) The implantation period with last 1-3 days C) I will have bathroom privileges D) I would have thought twice about pursuing treatment if I was not a mother with two toddlers

C - The patient will be on strict bedrest during the radiation implants. The patient will have a urinary catheter in place, and will get an enema before inserting the implant

Which of the following patients with skin disorders can benefit from ultraviolet {UV} light therapy? A) A 33 year old pregnant woman with eczema B) A mania patient with Alibert-Bazin syndrome C) A 22 year old Crohn's disease patient with psoriasis D) A patient post intraocular lens replacement surgery with skin cancer

C - UV light therapy is not contraindicated with Crohn's disease; {A} UV light therapy is contraindicated w pregnancy, {B} these patients are usually on antipsychotic meds which can cause photosensitivity, {D} UV light therapy is contraindicated with lens replacement

A patient is scheduled to receive an IV antibiotic q 8 hours, next dose is at 2p.m. The patient is prescribed peak and trough blood levels. At which time does the nurse schedule the trough level to be drawn? A) 1430 B) 1500 C) 1330 D) 1400

C - a trough level is drawn approximately 30 minutes before the next scheduled dose

A patient posts an open- reduction internal fixation (ORIF) for a fractured right femur. The Nurse can't palpate the pedal pulse in the right leg. Which action should the nurse take next? A) Check the pain score using a numeric scale B) Assess the right foot for redness C) Assess for acute compartment syndrome D) Put the patient in a supine position and raise the right leg on a pillow

C - absence of pedal pulse indicates acute compartment syndrome

A 45 year old patient with insulin dependent diabetes mellitus. The patient is asking if he should take the yearly flu vaccine. Which response by the nurse is appropriate? A) Only healthcare providers need to get the flu vaccine every year B) The influenza vaccine is required every other year for a patient with chronic medical illness C) The inactivated influenza vaccine should be taken yearly D) Stay away from the influenza vaccine because it might give you the flu

C - an annual or yearly influenza immunization is recommended for everyone over the age of 6 months especially a patient with chronic illness

The nurse is caring for multiple patients. Which patient will the nurse assign to the LPN? A) Patient who had an accidental pneumothorax during appendectomy B) A MVA patient with high ICP who is not easily arouse C) A patient with cellulitis on 2nd dose of antibiotics D) A patient for discharge who had a fractured fibula with an external fixator

C - antibiotics for cellulitis is the most stable patient; LPNs can not take care of unstable patients, RNs must do admissions and discharges

The son of an Alzheimer patient decided to take his father in to live with him and his wife. Which observation concerns the nurse? A) Electrical extension cords properly attached to the walls B) There is an analog clock on the hallway wall C) There is a blow-dryer hooked in the bathroom D) Locks are present on the top of the doors

C - any electrical devices should be secured to prevent a safety risk to the patient

Which item does the nurse use to test the oculomotor nerve? A) Coffee B) Cotton C) Torch D) Salt & vinegar

C - assess PEARL - pupil size, equality, accommodation reflex, and reactivity to light; {A} coffee is smelled to test the olfactory CN1, {B} cotton is touched to cheek to assess trigeminal nerve CN5, {D} salt & vinegar checks taste the CN7 facial nerve

A patient with chronic bronchitis and emphysema is admitted to the ICU for treatment. The patient receives IV aminophylline continuous infusion into a central catheter. Which adverse effects should the nurse be aware of and monito during the infusion? A) Bradycardia and hypertension B) Hypoactive bowel sounds & constipation C) Seizure activity & heart arrhythmias D) Dysuria

C - assess for hypotension, arrhythmias, and convulsions until aminophylline serum levels stabilize within therapeutic range

The debates over patient's abortion rights are often based on conflict between which pairs of ethical principals? A) Restitution and fidelity B) Justice and veracity C) Autonomy and nonmaleficence D) Paternalism and beneficence

C - autonomy is the freedom to decide the right to refuse, nonmaleficence is the unintentional action that causes harm

After a swimming accident, the patient became paraplegic. The patient experiences autonomic dysreflexia. Which response by the nurse is appropriate to assess this condition? A) Assess for diarrhea B) Assess fluid intake C) Check when was the last time the indwelling catheter was irrigated D) Assess for any symptoms of respiratory depression

C - bladder distension creates a noxious stimulus, which causes the potentially life-threatening complications experienced by victims of spinal cord injury. Autonomic dysreflexia can be caused by intestinal or urinary obstruction.

The nurse is giving discharge instructions to a 10 year old boy with a fractured radius bone. Which observation requires the nurse to refer the patient to home health? A) The boy didn't go to the playroom during the hospital stay B) The father works night shifts C) The boy's mother has bruises around her neck D) The mother is anxious to leave the unit

C - bruises around the neck could be signs of abuse at home. Violence in the family is a rsik factor for child abuse

A diabetic patient just came back from the post-anesthesia care unit after a transurethral resection of the prostate. Which intervention will the nurse perform first? A) Perform a bedside bladder scan to check for urine retention B) Collect urine for culture C) Assess the patency of the indwelling urinary catheter D) Obtain a glucose finger stick

C - check the patency of the foley catheter immediately after the procedure; glucose stick is needed for the diabetic, assessing the risk of clot formation is a higher priority

The home care nurse draws blood work for a patient diagnosed with schizophrenia. The patient is on Fluoxetine daily. The next morning the results come back with Na+ 127. Which action should the nurse take? A) Transfer to the hospital and initaite suicide precaution B) Get ready to start IV hydration of 0.9% NS C) One on one observation and initiate seizure precautions D) Do a thorough neurological assessment & report to doctor

C - critically low levels of sodium require immediate evaluation due to the risk of seizures

A patient was just put on mechanical ventilation for respiratory distress. Which is the highest priority parameter for the nurse to check after putting the patient on the machine? A) Respiratory rate B) Tidal volume C) Blood pressure D) Alarm parameters on the ventilator

C - decreased cardiac output is a complication of mechanical ventilation due to positive pressure within the chest. The RR, tidal volume and alarms are preset on the ventilator before putting the pt on.

What is the cause of high airway pressure alarms on a mechanical ventilator? A) Cuff leak B) Tachypnea C) Decreased lung compliance D) Self extubation

C - decreased lung compliance; Cuff leak and self extubation will cause low exhaled volume/ pressure airway, tachypnea will set off the high minute ventilation alarm.

A patient calls the doctor's office reporting fever of 101 F and burning sensation in his urine. As per protocol the nurse instructs the patient to collect a urine sample for culture. Which statement by the patient indicates teaching is successful? A) Call the lab 1 hour before delivering the specimen B) Will drink 1 liter of water before giving the sample C) I will use an aseptic technique to collect the urine sample D) I will collect an afternoon sample

C - decreasing the risk of contamination of the urine sample; drinking 1 liter of water will dilute the urine and may cause inaccurate urinalysis

The RN is educating the patient on the use of an incentive spirometer. Which instructions does the nurse include? A) Maintain spirometer at 45 degrees angle while breathing in B) Blow into the spirometer for 3 full seconds C) Deeply inhale through spirometer and hold your breath for 3 seconds D) Maintain the spirometer straight up to allow the aerosols to drop into the lungs

C - deep inhalation and holding breath for 3 seconds prevent atelectasis; should be eye level so that they can watch the level of intake, spirometer does not have aerosol - thats an inhaler

A psychiatric patient on the unit continuously tells the nurse that his stomach is missing. Which response by the nurse is appropriate? A) Losing weight should not be a problem for you then B) When do you think it went missing C) Sounds like you feel very lonely here D) I will do whatever it takes to help you get better

C - delusions often reflect the patient's underlying emotion. The nurse should first respond to emotions then orient the patient to reality. Don't encourage the delusions

Which of the following is not an exocrine gland? A) Parotid gland B) Gastric gland C) Pituitary gland D) Gallbladder

C - exocrine glands secrete fluids

Which question is most important for the nurse to ask when screening a patient for sleep apnea? A) Do you frequently wake up at night? B) Do you struggle to sleep? C) Does your partner complain of your snoring? D) How many naps so you get during the day?

C - first sign of sleep apnea is snoring

An enema is given to a patient with impacted bowel. The patient's heart rate drops from 80-50 beats per minute once the nurse starts introducing the enema. Which action does the nurse take first? A) Withdrawal the rectal tube B) Reduce the flow rate of the enema C) Stop the enema D) Position the patient on his left side

C - first step is to stop what is causing the vagal stimulation, then put patient on his left side, remove the tube and reassess. Later you may be able to try again at a slower rate.

A student nurse is assessing a 60 year old patient for dehydration under the supervision of the nurse. When would the nurse intervene? A) Assess orthostatic blood pressure B) Checks the trend of patient's weight over the last week C) Assess the skin turgor on the thigh D) Checks the electrolyte level

C - in this patient population, it it best to check the skin turgor by pinching the skin on the sternum or the hand for accurate assessment

Which is an indicator that the patient had an effective intervention to treating her mild pre-eclampsia? A) Blood pressure of 146/82 mm Hg B) Periorbital edema C) Deep tendon reflexes of 2+ D) Proteinuria of 3+

C - indicates that the intervention is working

A 47 year old female patient is getting ready for discharge after being treated for nephrotic syndrome. Which instruction is most important for the nurse to include in the discharge teaching? A) Monitor your blood pressure twice a week B) Switch to a low fat diet C) Signs and symptoms of thromboembolism D) Implement low sodium in your diet

C - is a very common complication; {A,B,D} all are important and true but C is the most important

The patient is currently on streptokinase for a recent stroke. Which lab value would cause the most concern? A) Serum glucose of 187 B) INR of 1.3 C) Platelet count of 85,000 D) Hemoglobin level of 8.4

C - low platelet count has a higher risk for bleeding

Patient is getting gentamicin IV every 8 hours. The provider ordered a gentamicin peak. If the medication is administered at 10 am over one hour, at which time should the nurse draw the gentamicin peak? A) 10 am just before giving the dose B) 12 noon C) 11:30 am D) 5:30 pm just before the next due dose

C - peak occurs 30 minutes after to completion of IV drug

A 50 year old patient who just had a heart attack. He is prescribed enteric coated aspirin. Which action does the nurse take in this situation? A) Administer the aspirin with a lot of water via nasogastric tube. B) Obtain a blood type and crossmatch for transfusion. C) Check his past medical history for peptic ulcer disease D) Get a 6 lead electrocardiogram prior to administering the aspirin

C - peptic ulcer disease is a contraindication for non-enteric coated aspirin {normal can cause peptic ulcers}. Enteric coated meds can not be crushed & given through NG

The patient tells the nurse he is having urinary incontinence 3 days after his prostatectomy. He is changing incontinence pads. Which action by the home health nurse is appropriate? A) Instruct the patient to drink less fluids B) Encourage the patient to use artificial sugar in his drinks C) Encourage the patient to perform pelvic muscle strengthening exercises D) Take doxazosin 2 mg PO daily

C - performing the exercise will enhance bladder control. The exercises should be done several times a day and will see improvement in a few weeks.

A 66 year old patient has been diagnosed with Diabetes Mellitus type 1 for 10 years. The patient is at risk for left big toe amputation. What is the best goal the nurse would set for the patient's plan of care? A) Normalized cholesterol levels B) Take safety measures while riding his motorcycle C) Maintain good glycemic control D) Perform daily range of motion exercises

C - peripheral vascular disease is prevented with optimal serum glucose control

Which of the following is not a sign of SLE? A) Fever B) Nephritis C) Photophobia D) Hypertension

C - photosensitivity is a sign of SLE

A 60 year old patient is admitted to the med surg unit for pneumonia. The nurse is developing the plan of care for the patient, which intervention is appropriate in the process? A) Using a generalized nursing care plan B) Plan to transfer to a nursing home C) Assess patient's baseline functional abilities D) Encourage physical activity and mobility during hospitalization

C - pre admission baseline assessment is important for comparison

A patient who is experiencing domestic violence asks the nurse why she is continuously getting beaten. What response is best by the nurse? A) Do you remember why you are being hit? B) Let us concentrate on your healing process now C) You don't deserve the abuse, we are here for you whenever you are ready for the help D) Your boyfriend is the only one who can answer why he is beating you

C - reinforcing the patient's worth and stating that help is available is therapeutic; A- never imply they deserved the abuse, B- do not delay emotional healing, D - is not therapeutic

A patient is experiencing oral and esophageal mucositis after his last chemotherapy treatment. During the discharge teaching, which comment by the patient indicates successful teaching? A) I will stop by the grocery store on the way home to pick up a mouthwash for gargling B) I will swish and spit my nystatin after every meal I eat C) I will chill down this week and play with my legos D) I will drink a glass of lemon juice with black pepper every morning to help heal the lacerations in my mouth

C - relaxation helps boost the immune system, distraction from pain is important ; ;Commercial mouthwashes contain a high concentration of alcohol & that will not help heal their mouth, the patient should swish and swallow nystatin, lemon juice is citric and is counterproductive to the healing process

The nurse plans care for a 60 year old patient. Which intervention does the nurse implement to reduce this patient's risk for falls? A) Raise the bed to the waist level B) Encourage patient to wear socks when walking the hallway C) Place the bedside commode close to the patient's bed D) Bring the chair and the food table closer to the patient's bed

C - rushing to the bathroom is one of the highest risks for falls. Put the bed at the lowest height, encourage pt to wear NON-skid socks, D may cluster the area with both the chair and food table increase the risk of falls

The doctor prescribes sulindac 150 mg PO twice a day for 10 days. Which symptoms should the patient report immediately to the doctor? A) Anorexia B) Double vision C) Ecchymosis of the lower limbs D) Vomiting in the morning

C - sulindac is a nonsteroidal anti-inflammatory drug, patient should inform doctor if bruising or bleeding occurs

Which of the following is not a side effect of ACE inhibitors? A) Proteinuria B) Neutropenia C) Tachycardia D) Skin rash

C - tachycardia is a side effect of calcium channel blockers

The student nurse is performing a dressing on a PICC line while the preceptor is supervising him. After removing the dressing, which action by the student requires the preceptor to intervene? A) Use the Iodine to clean the catheter insertion site B) Cover the catheter with a 4X4 gauze to get his sterile field ready C) Using a circular motion, he cleans the catheter site from the outside towards the insertion site D) Place an occlusive dressing and marking the date on it

C - the cleaning motion is circular from the inside to the outside

The nurse is leading a grief support group at the mental health center. Which patient is not ready to enroll in this group yet because additional assistance is needed? A) 45 year old patient who spouse died 3 months ago from a heart attack B) A 55 year old female who lost her husband 3 years ago due to ALS C) A 38 year old female who started drinking 4 months ago just after her husband died by a car accident D )A 25 year old male who lost his wife 9 months ago to brain cancer and is left with two toddlers

C - the drinking problem needs to be addressed first in order to resolve the grief

The charge nurse is distributing the assignment for the nurses on the night shift. Which patient is most appropriate to be assigned to the LPN? A) A psychotic patient B) A patient undergoing chemotherapy administration C) A patient who has right leg skin traction D) A patient undergoing blood transfusion

C - the most stable patient with expected outcomes; Things to ask: Is this pt stable? Does this pt have outcomes (are they high risk)?

A patient states: I promise not to touch the foley anymore, I don't want to be slapped. Which action does the nurse take first? A) A thorough neurological assessment B) Questioning the nursing assistive personnel if the patient was slapped when providing care C) Ask the patient where he got slapped and who did it D) Document the patient's exact words and inform your supervisor

C - the nurse has to assess if an assault or battery occurred and by whom. This should be done prior to documentation and reporting the event to your manager. File an incident report no documentation in pt chart is needed.

The pediatric nurse is taking care of a 3 year old baby. The nurse is using play therapy to distract the child. Which toy or activity is most age-appropriate? A) Playground materials such as a playhouse B) Housekeeping toys like a cleaning trolley C) Stuffed animals D) Coloring books

C - the nurse should always be aware of patient's safety & carefully screen toys for safety concerns ; the other options are those for a preschool age child

The charge nurse observes the nurse delegate a dressing change for a patient with fever, neutropenia, and hypotension to the LPN. Which action will the charge nurse do next? A) Allow the LPN to complete the assigned task B) Ask another experienced LPN who is more comfortable with dressing changes to complete the task C) Explain to the nurse that the dressing change can not be delegated to the LPN D) Ensure that the nurse follows up with the LPN if the task was done

C - the patient is not stable and the nurse should complete the dressing change instead

The nurse at a community center is screening people for gastric cancer. What is the appropriate question for the nurse to include in the screening survey? A) Any history of ulcerative colitis? B) How often do you have energy drinks? C) Any history of Helicobacter pylori infection? D) Are you into wine drinking?

C - the presence of H. pylori in the stomach increases the risk for gastric cancer. H.pylori causes peptic ulcer disease; ulcerative colitis puts the pt at risk for colon cancer

During an actual disaster. Which patient would the nurse assess first? A) An infant with a cut on the forehead, presenting a SBP of 86 and a heart rate of 110 beats per minute B) A child with a compound fracture in the right femur, dilated pupils, and no pulse C) An adult with a tourniquet applied to the right thigh, below knee amputation, a SBP of 81 and HR of 130 beats per minute D) An adult crying non-stop with bruises on the forehead and a GCS of 13

C - the vital signs could indicate that the patient who has a below knee amputation could be going into hemodynamic shock. This patient should be seen first

During the nurse's initial assessment of a new admission to the med surg unit. What observation indicates that the patient has a history of alcohol use? A) Anorexia, difficulty concentrating, depression B) High SGOT, cirrhosis on abdominal ultrasound C) Tremors, hyperthermia, leg cramping, generalized pain D) Headache, runny nose, night sweats, hyperthermia

C - these are symptoms associated with an alcohol-related problem {A} symptoms are signs r/t mood disorders; {B} can be caused by alcohol use but is not an indication of alcohol use {D} can be signs of withdrawal of narcotics not alcohol

A patient who had gastric surgery 2 days ago has an NGT attached to intermittent suction. The patient had a large amount of secretions in the last day down in the suction bottle. Which ABGs result will the nurse expect to see? A) pH - 7.23, PaCO2 - 48 mmHG B) pH - 7.39, PaCO2 - 38 mmHg C) pH - 7.49, HCO - 30 mmol/L D) pH- 7.28, HCO - 20 mmol/L

C - this ABG reflect metabolic alkalosis

The mother of a child with tetralogy of Fallot is pumping her breast at the bedside. The unlicensed assistive personnel (UAP) criticizes the mother in front of the nurse and is annoyed that the mother is not breast feeding the child instead. What is the best response by the nurse? A) You look upset, tell me more about your childhood B) Did you ask the mother the reason she is using the breast pump C) What do you know about the child's defect? D) You are not here to judge others about their actions

C - this is a good starting point to educate the staff about cardiac disorders. Patient with TOF needs the least amount of effort to feed. You want to remain professional and educate your colleagues about

A 56 year old patient was transferred to the ED after wandering away from home. The patient was found confused and restless. Which action is important to include in the patient's plan of care? A) Accept patient's behavior B) Give the patient's time to get used to the nurses C) Maintain a stable and consistent environment D) Make decisions on behalf of the patient until his confusion is gone

C - this is the most beneficial to reduce anxiety of the new space

Which order does the nurse question for a patient currently on a telemetry monitor? A) A CHF patient on dobutamine drip who is prescribed a new order of continuous IV Morphine B) A newly diagnosed Atrial fibrillation patient with creatinine level of 1.3 is prescribed 40 mg of Enoxaparin BID C) A Patient 6 hours post CABG has 1200 ml of serosanguineous drainage in his chest tube, is prescribed 20 mg of enoxaparin twice daily D) A patient post cardiac catheterization is prescribed coumadin while still on IV heparin drip

C - this pt is draining 200 ml per hour, if chest tube drainage has more than 100 ml per hour the HCP should be informed, enoxaparin is contraindicated in bleeding situations ; Dobutamine & morphine do not interact, heparin should still be given while starting coumadin {it takes several days to get to a therapeutic level} monitor and slowly drop dose of heparin.

A patient is admitted after a MVA, the patient has a body mass index of 17. Which assessment will the nurse identify as a health problem? A) Hydromorphone allergies B) Arthritis C) Pressure sore injury D) Low blood pressure

C - underweight patient would negatively impact wound healing, {bony prominences}

A 20 year old man looking for a gift idea for his father who is diagnosed with Parkinson. What would be the best option? A) Hair product and grooming clippers B) A key chain for the home keys C) Swivel utensils for food D) A modern foam mattress

C - useful assistive devices for Parkinson patients may include utensils with large, rubber grips, swivel utensils that are designed to counterbalance to the patients' tremor and spill proof cups

The nurse is preparing to give a medication IV push into an established peripheral IV line. Which action does the nurse take? A) Choose the distal port to the IV cannula B) Open the IV to a free flow rate while giving the medication C) Time the medication administration process D) Explain what just happened to the patient

C - using a watch to record the time of administration ensures safe drug infusion; Use the proximal port, use a pinch and release method on the IV line to prevent back flow and flush the drug, educate the patient before you administer meds

A patient with peripheral arterial disease complains during his clinical office visit that he is experiencing leg pain during long walks. Which statement will indicate that the patient understands proper interventions? A) I will lie down on my back and elevate my feet when I experience the pain B) I will apply a heating pad to my legs when I experience the pain C) I will walk until I experience pain. I will rest, and then resume my walk D) I will perform dynamic stretching of my muscles for 10 minutes before and after the walk

C - working the muscle will demand additional flow of blood to the muscle to provide oxygen and remove the wastes

What is the best indicator that a patient with anorexia nervosa is responding to treatment and met the goal of balanced nutrition? A) Patient is loving the way she looks B) The patient is eating balanced meals C) The patient's menstrual cycle is back to normal D) The patient reached an ideal body weight

C -Amenorrhea is a sign of anorexia nervosa, a normal cycle shows that the pt has met goal

The nurse provides care for a patient who had an unexpected death during a night shift. The patient has many tubes and drains in place. The nurse is performing postmortem care of the deceased patient. Which action by the nurse is appropriate when managing the tubes prior to a scheduled autopsy? A) Discontinue tubes and drains and send to autopsy with the body B) Discontinue tubes and drains and put them in biohazardous bags C) Keep all tubes and drains in place in the patient's body D) Keep Iv tubes in place but remove drains.

C -If an autopsy is to be performed, any tubes or drains will be left in place to be assessed and cultured, then removed by the medical examiner

A patient right after post hypophysectomy. He is thirsty and has frequent urination. Which action does the nurse take first? A) Check the glucose in urine B) Encourage drinking water C) Check urine specific gravity D) Call the surgeon

C -Post-op, diabetes insipidus can temporarily occur due to low ADH

The nurse creates a dietary teaching plan for a pregnant patient. Which information will the nurse include? A) Protein requirements will double B) Increase calories by 800 kcal/day C) Need to increase iron D) Decrease sea salt intake

C -a pregnant patient requires 27 - 30 mg/day of iron, which would be hard to get only from diet. The pregnant patient may be prescribed iron supplements starting on the second trimester; you only increase protein by 30 g/day

The nurse is assigned to take care of 4 patients, after reviewing their history which patient is identified as being most at risk for developing breast cancer? A) 29 year-old nulliparous patient B) 45 year old patient with a familial history of breast cancer C) 58 year old patient with history of breast cancer at 38 D) 60 year old patient who has been menopause for 13 years

C most at risk- age greater than 50 is at risk for developing BC especially with a positive hx; First pregnancy is at 30 years old ^ risk for breast cancer, pt with family history of BC is at risk, women over the age of 50 are at risk for breast cancer

The patient is scheduled for surgery the next morning. The order states NPO without mentioning medication. Which medication causes the nurse to question its administration prior to surgery? (SATA) A) Atenolol 25 mg for hypertension B) Midazolam 5 mg to prevent seizures C) Aldosterone 25 mg for hypertension D) Dexamethasone 4 mg for COPD E) Clopidogrel 75 mg given daily for a-fib F) Alprazolam 2.5 mg given at bedtime for insomnia

C, E - You don't hold antihypertensives before surgery unless it is a diuretic (aldosterone), give midazolam so that the levels are not messed up and cause seizures, clopidogrel is a blood thinner

The nurse provides care for multiple patients. Which tasks does the nurse delegate to the nursing assistive personnel?(SATA) A) Assess patient's pain level B) Teach patient how to perform crutches use C) Hand meal trays D) Give bath to a patient with a surgical site E) Get standard vital signs

C,D,E

The nurse intervenes when the patient takes licorice with which other medications? A) Promethazine B) Ibuprofen C) Potassium chloride D) Furosemide E) Prednisone

C,D,E - Potassium chloride is taken for hypokalemia, licorice has a hypokalemic effect -counterproductive; furosemide also lowers K and can cause hypokalemia; Licorice will increase the effect of corticosteroids

The nurse assesses a patient diagnosed with colorectal cancer. What symptoms does the nurse ask the patient during admission assessment? (SATA) A) Instant abdominal bloating B) Severe spasmodic abdominal pain C) Ribbonlike stools D) Rectal bleeding E) Diarrhea and/or constipation

C,D,E - these are symptoms of colorectal cancer. Patients with colon cancer, especially those with cancer on right colon may experience colics, cramping and pain, left of colon {colorectal cancer}will not; Instant abdominal bloating is a sign for obstruction

A patient is diagnosed with DVT. For which symptom does the nurse notify the health care provider? A) Lower back pain B) Ankle and elbow soreness C) Chest tightness D) Restlessness at night

C- Chest tightness or discomfort might be the earliest sign of a PE. Should be assessed and reported immediately

Which Outcome indicates that the intervention for a patient with syndrome of inappropriate antidiuretic hormone {SIADH} has been effective? A) Decreased serum osmolality B) Serum sodium is decreased C) Increased urine output D) Increased urine osmolality

C- Increased urine output - sign that treatment is effective

A patient had a car accident, is unconscious from a severe head injury and a back fracture. The patient has no ID on him, but needs emergency surgery. Which action is best for the nurse to take when obtaining informed consent for the surgery? A) Ask the ED provider to sign the informed consent B) Get an urgent court order for the surgery C) Transport the patient to the OR for surgery D) Request the police to locate the family and identify the patient

C- Informed consent of an adult is not needed when in emergency situation, and delaying treatment could cause death of patient

The nurse just finished getting a shift report. Which patient will the nurse see first? A) 24 hours post appendectomy patient B) 4 hours post cataract repaired patient C) A patient with an esophagogastric tube inserted during the morning shift D) A patient 2 days post spinal fusion surgery

C- Monitor BP & Hr to assess for hypovolemic shock. Esophagogastric/Blakemore tube is indicated for gastric bleeding especially with Cirrhosis

The nurse is participating in a community program for victims of sexual abuse. What statement by a victim indicates a positive response to therapy? A) My actions lead to me being sexually abused B) The trust relationship with my father is reestablished C) I am not responsible for the abuse D) I still feel anxious sometimes when talking about my abuser

C- Realizing and verbalizing that the abuse was not their fault is the first step in the healing process

Patient had a car accident, she is oriented to self but not to time and place. The patient complains of a severe headache and is restless. Which action does the nurse take first? A) Help the patient remember the current location and time of the day B) Put the patient in a vest restraint C) Neurological assessment every 15 minutes D) Give the patient PRN morphine and lorazepam

C- Remember ADPIE, increased ICP is suspected. Confusion, restlessness, pupillary changes, and altered level of consciousness are the earliest signs

The nurse provides care for a patient with facial, and neck burns. What is the best position for the patient? A) Prone with a rolled sheet under the head B) Supine with padding on the affected side C) Supine without pillows or padding D) Prone without extra padding around the head

C- a supine position minimizes pressure and irritation to the burned areas, but burned areas may stick to pillows or padding. Pillows may cause contraction in patients with neck burns.

The nurse provides care for a 16-year old patient who is diagnosed with meningitis. The provider ordered intravenous and oral hydration. The nurse closely monitors the patient's fluid intake. Which serious complication does the nurse monitor for this patient? A) Heart failure B) Hypovolemic shock C) Cerebral edema D) Pulmonary edema

C- because of the inflammation of the meninges, the patient is vulnerable to developing cerebral edema and increased intracranial pressure.

Which patient would the antepartum nurse assess first? A) A patient with epistaxis at 10 weeks of gestation B) A patient who just noticed rectal varicosities at 14 weeks of gestation C) A patient complaining of epigastric pain at 20 weeks gestation D) A patient complaining of leg cramps at 34 weeks of gestation

C- continuous or intense abdominal pain can be an indication for ectopic pregnancy, preeclampsia, or abruption placenta; Nosebleeds are normal during pregnancy, large uterus puts pressure on the rectum and can cause varicosities, leg cramps are common in the third trimester

The nurse assess distended neck veins in a patient reporting shortness of breath and chest pain. Which problem is causing the Jugular Vein Distention? A) Dehydration B) Brain mass C) Fluid overload D) Electrolyte imbalance

C- fluid overload causes increase of blood volume. This increase causes the veins to distend, most obviously in the neck veins.

A patient has severe hypothermia 14 hours post mitral valve replacement. Which assessment will the nurse perform first? A) Check if the patient is shivering B) Determine if the patient is cyanotic C) Check telemetry monitor for arrhythmia D) Determine if calcium is low

C- hypothermia puts the patient at risk for ventricular dysrhythmia

A patient experiences a fever, headache, photophobia, and neck stiffness. Which transmission-based precaution will the nurse implement for this patient? A) Contact B) Airborne C) Droplet D) Standard

C- the patient's symptoms are consistent with N. meningitis and droplet precautions should be used

Nursing report was given to the night shift nurse about four pregnant patients in active labor. Which patient does the nurse assess first? A) A nulliparous patient 10 cm dilation and 100% effacement B) A nulliparous patient with fetus in transverse lie and FHR of 155 BPM C) A multipara patient at 8 cm dilation with the presenting fetal part at 2+ station D) A multipara patient at 0 station with fetus in breech presentation

C-most multipara women have very fast transition phase; D - the fetus is still in breech position, B - the fetus is stable and in transverse lie; A- even though they are fully dilated and effaced, nulliparous women take longer to transition

The SpO2 of a lung cancer patient drops from 92% to 85% during ambulation. Which action does the nurse take next? A) This is a normal drop in response to activity with lung cancer patients B) Relocate the oximetry probe to the earlobe during activity C) Obtain an order for supplemental oxygen to be used during activity D) Obtain an order for ABGs to verify the arterial oxygen saturation

C-supplemental o2 is needed to increase supply of oxygen during activity; the probe was reading well at 92%, no need to relocate

A 55 year old woman is admitted to the med surg unit for back pain. The nurse is educating the patient about bone health. Which information is crucial for the nurse to include in her teaching? A) One of the major risk factors is being an African American B) Staying in a long term facility for more than 6 months C) High fat diet D) Being on steroids for a long time

D

During physical assessment, the nurse checks the patient's back and sees small pinpoint hemorrhagic dots. What does the nurse document in her chart? A) Extravasation B) Cyanosis C) Melanoma D) Petechiae

D

What is the best position for a patient in COPD exacerbation? A) Prone B) Semi fowler C) High Fowler D) Tripod position

D

Which of these food selections are not rich in iron? A) Fish B) Legumes C) Whole grains D) Chicken

D

The triage nurse gets a message about a patient diagnosed with Parkinson. The patient is having difficulty with his speech. What should the nurse be thinking about? A) Initiate facial muscle exercise B) Depression among Parkinson patients leads to speech issues C) Will require to change his medication regimen because of disease progression D) This finding puts the patient at risk for aspiration

D - Because the muscles for speaking are the same as those used swallowing, the nurse determines that this patient may be at risk for aspiration

The nurse instructs a patient receiving olanzapine. Which statement made by the patient to the nurse requires further teaching? A) "This medication will help my thoughts and behavior" B) "I must report restlessness to the health care provider" C) "I will tell the health care provider if I am planning to get pregnant" D) "Stiffness and tremors are expected for the first 2 weeks"

D - Stiffness and tremors are extrapyramidal symptoms and are reported immediately. This statement indicates a need for further teaching

A patient with HIV is experiencing a new abnormal skin tissue, the physician is suspecting a cutaneous Kaposi sarcoma. Which assessment confirms the diagnosis? A) Swelling and blockage of the lymphatic system B) Slightly elevated purple lesions all over the skin C) Swelling in the genitalia D) Punch biopsy of the lesion

D - a punch biopsy will provide an accurate diagnosis

The nurse audits the documentation of the student's assessment of the patient's anterior chest. The student has correct documentation when the nurse sees what? A) Excursion of the diaphragm is 3 cm B) Asymmetric expansion of the chest C) Clear vesicular breath sounds over bilateral lungs D) Symmetric ribs and intercostal spaces noted

D - appropriate inspection of the anterior chest; Excursion of the diaphragm & expansion of the chest is assessed on the posterior, breath sounds are assessed posteriorly & anteriorly

The patient's wife who is diagnosed with Alzheimer asks if the new medication will improve the patient's dementia. Which response is appropriate by the nurse? A) The patient will be able to live independently B) The progression of Alzheimer will stop C) It will help improve the short term memory D) It will control symptoms of dementia but will not improve it

D - dementia is irreversible

The nurse sees another nurse don a gown, gloves and face mask before entering a patient's room. What would the nurse expect the patient's diagnosis to be? A) 1 year old with RSV B) A 6 year old with hepatitis A C) A 15 year old with C. Diff D) A 20 year old with influenza A

D - droplet precaution is needed

A 28 year old patient has been having nausea and vomiting for a couple of weeks now. The nurse performed a pregnancy test and it came back positive. By calculating the last menstrual cycle, the patient is 10 weeks pregnant. Which assessment finding does the nurse expect to see? A) Linea nigra on the belly B) Chloasma On the forehead C) Leg cramps D) Hypertrophy of mammary glandular tissue

D - enlargement of the breasts is caused by hormonal stimulation during the start of the pregnancy

A patient is on phenytoin for his idiopathic seizure disorder. The patient is going on a date in 2 weeks and is concerned about having an episode. Which action is the best for the nurse to take? A) Call the doctor and suggest increasing his phenytoin dose on the day of his date B) Attend the date to provide assistance C) Go over some relaxation exercises that the patient can practice on his date D) Encourage the patient to have a seizure diary

D - having a dairy will help monitor what the triggers are and avoid those activities

The patient is taking two tablets of potassium (20 meq) BID. The patient's creatinine level is 1.7 mg/dl. What action is the priority for the nurse? A) Ask for a nephrologist consult B) Give the potassium as scheduled C) Assess the patient's fluid balance D) Inform the provider

D - high creatinine level could be a result of imparied kidney function thus, result in hyperkalemia. The priority for the nurse is to notify the provider before the dose

A 24 year old patient just gave birth and was transferred to the unit. Upon arrival, she asked the nurse for 20 pads and at least 15 diapers. What response by the nurse would be best? A) I will be right back with what you asked for B) You will be discharged in 4 hours, why don't you stop by the store on the way home. C) That's too many supplies D) Just tell me what items you need for the next hour

D - is a professional response that is still getting to the patient's needs but in a cost effective way

A patient was admitted though to ED complaining of right sided weakness and trouble speaking. The patient should be maintained in what position? A) Sims recumbent position with the head of the bed elevated 30-40* B) Right side position, with the head of the bed elevated 30* C) Left side position with the bed flat D) Supine position, with the head of the bed elevated to 30*

D - maintain the head in supine position & help decrease patient's ICP

A new mother tells the nurse that she wants to exclusively breastfeed her newborn. The nurse reviews newborn nutrition with a mother. Which statement by the mother indicates successful teaching? A) At 4 months of age, I will start mixing baby cereal in the milk bottle to give my baby girl B) I will throw away any breast milk in my freezer that is more than 4 months old C) I will introduce baby cereal at 3 months of age to help with the growth D) My baby girl has enough iron stores from my breast milk that will last her 4 months

D - one nutrient that is present in breast milk is iron ; {A} baby cereal will not mix in the milk bottle, {B} breast milk can last up to 6 months in the freezer, {C} introduce the baby to solid food around 4-6 months

The nurse calls the mother of a patient diagnosed with antisocial personality disorder to follow up on the patient's progress with therapy. The mother states that her child looks way better and is accepting greater responsibilities. Which response by the nurse is best? A) That is awesome, I am happy to hear that B) Statistics show that children with antisocial disorders do not drastically change C) Continue the medication and therapy even if you see improvement D) Walk me a little bit through the new responsibilities your child has been taking

D - open ended statements give the opportunity to further assess the situation

The Nurse is ready to discharge a patient diagnosed with open-angle glaucoma. What should the nurse include in their discharge teaching? A) Limit the movement of the eyes B) Wear sunglasses outdoors C) Instill tropicamide {mydriatic eye drops} BID D) Schedule tonometer checks twice a year

D - patients diagnosed with open angle glaucoma should have eye pressure measurements annually or biannually

The nurse supervisor is rounding on the psychiatric unit at the beginning of a night shift. What situation requires the supervisor to give immediate attention? A) The UAP removes the dinner tray of a patient diagnosed with major depression. The tray is still complete & untouched by the patient. B) The UAP is changing the bed linen while a bipolar patient is trimming his beard with an electric razor C) The LPN gets loud trying to convince a patient with bipolar disorder to take his medication D) The UAP puts some cologne on an Alzheimer patient after giving him a bed bath, places the cologne on the food table while patient is getting ready to eat dinner and goes to the dirty utility room to drop the used towels

D - personal care items impose choking risk for the patient with dementia


Conjuntos de estudio relacionados

Wong's Ch. 18: Impact of Cognitive or Sensory Impairment on the Child and Family

View Set

Alexander Zaiko Deca terms for HS test

View Set

Effective Police Supervision 8th Edition - Chapter 7

View Set

Financial Management LS Chapter 6

View Set

Cognitive Psychology Final (PSYC 2145)

View Set

NUR 221 EAQ - Chapter 2: The Family, Culture, Spirituality, and Home Care

View Set

Ch. 28 The Female Reproductive System

View Set